You are on page 1of 50

Ray Optics

* Light
" Reflection of Light
Spherical Mirrors
" Refraction
* Total Internal Reflection
.. Refraction from a Spherical Surface

..
,.
"

Lens
Refraction Through a Prism
Dispersion
Optical Instruments
~ Resolving Power of a Microscop~~
" Resolvi~g Power of Telescope

------------------,~.~------------------

t> light
The light is that form of energy which makes objects visible to our
eyes. The branch of physics which deals with nature of light, its sources,
properties, measurement, effects and vision is called "optics". For the sake
of convenience, study of optics is generally divided into two parts namely
(i) geometrical optics or ray optics, and (ii) wave optics. This chapter
deals with the geometrical optics.

*> Reflection of Light


When a beam of light is incident on a polished interface, it is thrown
back in same medium. This phenomenon is called reflection.
In reflection the frequency, speed and wavelength do not change, but
a phase change may occur depending on the nature of reflecting surface.
Experimentally it is found that the rays corresponding to the incident
and reflected waves make equal angles with the normal to the surface .
Thus, the two laws of reflection can be summarized as under.
1. Li = Lr

2. Incitlent ray, reflected ray and normal lie on the same plane.

886

Chapter 23 Ray Optics


Normal

J":

If

>bs0t-,

.(\iil Note
The above two laws of reflection can b.e appli.ed to the reflecting surfaces
wh}ch a(e not ev.en horizontal.

Instance 2 Find velocity of image when object and mirror


both are moving toward each other with velocity 2 ms-1 and 3 ms- 1
respectively.
(b) --Bms-1
(a) 8ms-1
(c) -5ms-1
(d) 5ms- 1
Interpret

Here

9rh r(g,.

Vo-VM=-(Vl-VM)

1t811ection from Plane Mirror (Surface}


In case of reflection from plane surface such as plane mirror
(i) The image is always erect, virtual and of exactly the
same size as the object. The image is formed as much
behind the mirror as the object is in from of it.
(ii) The image is laterally inverted.
(iii) If keeping the incident ray fixed, the plane mirror
is rotilted through an angle e, the reflected ray turns
through double the angle ie, 28 in that very direction.
(iv) If the object is fixed and the mirror moves relative to the
,object with a speed v, the image moves with a speed 2v
~elative to the object.
(v) If the mirror is fixed and the object moves relative to the
mirror with a speed v, the image also moves with the
same speed..y relative to the mirror.
(vi) Deviation suffered by a light ray incident at an angle i is
given by 8 = (180 - 2i) ,
(vii) If there are two mirrors inclined at an angle 8, the total
number of images formed for an object kept between
2
the two is equal to 7t or ( ~7t - 1} which ever is odd.
8
(viii) The minimum size of a mirror required to see the full
image of a person, is half the height of the person.
(ix) If a plane mirror is rotated about an axis perpendicular
to plane of mirror then reflected ray image do not
rotate.

Instance 1 The minimum size of the mirror fixed on the wall of a


room in which an observer at the centre of room may see the full image
of the wall of heightJLbehind him is
(a)

hx
z-(2x+y)

Size of mirror, M M -

(b)

(c) 2h

~
2

(d) h

lnterp.ret

From

~ 0' M1M2

M 1M 2 = - x -

2x +y

and O'AB

=> (+2ms-

1
) - ( -3ms- )

=~vi+ (-3)

Instance 3 Two plane mirrors are inclined at 30 as shown in


figure. A light ray is incident at angle 45. Find total deviation produced
by combination of mirror after two successive reflection.

(a)
(c)

60 i
50 i

(b) 58 i
(d) 68 i

Deviation at mirror M1 ,8 1 = 180-2 x 45= 90 i

Interpret

Deviation at mirror M2 ,8 2 = 180-2 x 15 = 150t


Total deviation 8 = 82 -8 1 = 150- 90 = 60 i

Spherical Mirrors
Mirrors having their reflecting surface spherical are called
spherical mirrors. Spherical mirrors are of two types
(i) Concave mirror If reflection takes place from the inner
surface, the mirror is called concave [Fig. (a)].
(ii) Convex mirror If reflection takes place from the outer
surface , the mirror is called convex [Fig. (b)] .
A
Incident
light

Incident
light

---+

-----...--....-=: P----.. +ve

---+

(a) Concave mirror

--=p~------

(b) Convex mirror

887

Chapter 23 Ray Optics

Definitions of Some Terms Related to Spherical Mirrors

Ray Tracing

Centre
and
radius
of
curvature The centre of curvature
and radius of curvature of a mirror
are the centre and radius of the
sphere of which the mirror is part.
In the given figure, AC is the radius
of curvature and C, the centre of
curvature.

In geometrical optics, to locate the image of an object.


Tracing of a ray as it reflects or refracts, is very important.
1. A ray going through centre of curvature is reflected back
along the same direction.

Concave

Convex

Pole Pole of the spherical


mirror is mid point of its reflecting
surface. In figure it is shown by P.

2. A ray parallel to principal axis is reflected through the


focus, and vice-versa. Also, mutually parallel rays. !\{!]1t.
reflection intersect on the focal plane.

Principal ,axis The principal axis of a spherical mirror is


the line joining the pole and centre of curvature. In the figure PC
is principal axis.
Principal focus Principal focus is a point on the principal
axis of the mirror at which the light rays coming parallel
to principal axis actually meet after reflection or appear to
meet.

.E)

3. The light corning through the focus of rriirror or coming


towards focus, becomes parallel to principal axis.
;)

c
F

(b) Convex mirror

(a) Concave mirror


0

For concave mirror focus is infront of the mirror, while for


convex mirror focus is behind the mirror. Focus of concave mirror
is real, while focus of convex mirror is virtual.

Sign Convention for Mirrors


According to the sign convention
(i) Origin should be placed at the pole (P).
(ii) All distances should be measured from the pole (P).
(iii) Object distance is denoted by u, image distance by v,
focal length by f and radius of curvature by R.
(iv) Distance measured in the direction of incident ray are
taken as positive while in the direction opposite of
incident ray are taken negative.

Focal length The distance between pole and focus of


a spherical mirror is called its focal length. It is represented
byf

ie,

Table 23.1

1.

= !!_

Image Formation by Concave Mirror

At infinity

Real inverted, very small [m < < -1], at F


t-

2.

Between infinity and

M'

Real, inverted, diminished (m < -1)


between F and C

888

Chapter 23 Ray Optics

3.

Real, inverted, equal in size [m = -1 ] at C

AtC

M'

4.

Real, inverted and very large (m > -1)


between 2F and iiliini.ty

Between F and C

M'

\
~

5.

Atf

Real, inverted, very large [m


infinity

6.

Between F and P

Virtual, erect, large in size (m > + 1)


behind the mirror

Table 23.2

1.

( -oo)] at

Image Formation by Convex Mirror

Virtual, erect, very small (0 < m < < + 1) at F

At infinity

M'

2.

Virtual, erect, diminished (m < + 1) between


PandF

In front of mirror

c
M'

Chapter 23 Ray Optics

889

\
Formula and Magnification for Spherical Mirrors

we have,
1

Mirror fonnula
1

-=-+! v u
where symbol possess their usual meanings.

Now,

I
v
f
f-v
m=-=-=--=-0 u f-u
f
where I = size of image perpendicular to principal axis
0 = size of object perpendicular to principal ruds.

Axial magnification

~~ = :: = ( f ~

=(f

~!

"

Areal magnification
2

-=----=-v l.Sf f
3f

Lateral magnification

max = -

-+-=-or--+-=-u v F
-l.Sf v
f

m = Ar =::.:_=(-!) =(f - v)
ar
Ao
u2
f- u
. f

ni

=-~"" __l_[_ =-2

u
l.Sf
h
or
__ = -2 or h2 = -2h1 = -5 em.
hl
The image is 5 em long. The minus sign shows that it is inverted.

Instance 5 A concave mirror of focal length 10 em and a convex


mirror of focal length 15 em are placed facing each other 40 em apart.
A point object isplaced between the mirrors, on their common axis and
15 em from the concave mirror. Find the position of the image produced
by the successive reflections, first at concave mirror and then at convex
mirror.
(a) 6 em
(b) +10 em
(c) + 15 em
(d) +30 (.7n
Interpret According to given problem, for concave mirror.
f= -10

where , A 1 = area of image


A 0 = area of object.

Instance 4 An object of length 2.5 em is placed at l.5f from a


concave mirror; where f is the focal length of the mirror. The length of
the object is perpendicular to the principal ads. Find the length of the
image. Is the image erect or inverted ?
(a) -5 em
(b) 5 em
(c) 6 em
(d, --6 em
Interpret
and

The focal length F


u

10

-f

= -l.5f

1--f

1 - - - -- 1.5 f ------;~

u=

15 em andf = - 10 em
.!.+-- = -1- ie, v = -30em
v -15 -10
ie, concave mirror will form real, inverted and enlarged
image 11 of object 0 at a distanci! 30 em from it, ie, at a distance
40-30 = 10 em from convex mirror.
For convex mirror the image 11 will act as an object and so for it
u = -10 em andf = + 15 em.
.!.+-1-=_.!._ ie,v= +6Cin
v -10 15
So, final image 12 is formed at a distance 6 em behind the convex
mirror and is virtual as shown in figure.

So,

lntext Que,~!lo~,. -2a.l M~'""

(i); Does the mirror fommla hold g?od for a plane mirrgr? . ,
,
. .. _._. _ _. :,:
, .....
(iit An object :is'placed between two plane parallel mirrors. Why do tpe
t images ;~t fainter .
(iii)' Why are inir:rors used in search-lights parabolic and not concave spherical?
, ; )>
qy~ If yo~ were driving a car, what typ~ ofmirtoi would .you prefer to use (9_r;ob~~~il:affic .at ""''"''"Adr?

w> Refraction
When light passes from one medium, say air, to another
medium, say glass, a part is reflected back into the first medium
andthe restpa~,;, .,
.:
.. ..;. n-''.': _,: ;\;)assesintu
the second medium, it either bends towards the normal or away
from the normal. This phenomenon is known as refraction.

laws of Refraction (Snell's law)


(i) If meditim 1 is a vacuum (or in practice air) we refer 1J.l2
as the absolute rE>frai'! h " ir><f : )f medium 2 and denote
it by J.l/ or simply J.l (if no other medium is there).

(ii) Now, we can write Snell's law as,

J.l sin i = constant


For two media, 11 1 sin i1 = J.lz sin i2

... (i)

... (ii)

890

Chapter 23 Ray Optics


Instance 7 A ray of light is incident on a transparent glass slab of
refractive index 1.62. If the reflected and refracted rays are mutually
perpendicular, what is the angle of incidence?

(iii) Snell's can be written as.


sin i1 v1
1112

A.
l-12
=-.- . - = - = 1- = Sllll2
v2 A. 2 111

Here, v1 is the speed of light in medium 1 and v2 in


medium 2. Similarly 1.. 1 and A- 2 are the corresponding
wavelengths.

(a) 58.3
Rarer

Denser

Denser

Rarer

;1 > ;2
V2

<

l-12

> l-11

ll :> < u l

"-2 < "-1

"-2 > "-1

VI

If 1-1-2 ::> 1-1-1 then v 1 > v 2 and A-1 > 1..2 , ie, in a rarer m~dium,
speed and hence, wavelength of light is more .
(iv) In general, speed of light in any medium is less than its
speed in vacuum. It is convenient to define refractive
index 11- of a medium as.

Interpret

Let the angle of incidence, angle of reflection and an,le


of refraction be i , rand r', respectively.
Now, as per the question 90 -r + 90- r' = 90
=>
r' = 90- i
(because i = r)
In case of reflection according to Snell's law, 1 sin i = 1-l sin r '
or sin i = 1-l sin (90 - i)
=>
tan i = 1-l
or
i = tan-1 [!1-] = tan-1 (1.62) = 58 .3

Instance 8 Refractive index of glass with respect to water is 1.125.


If the absolute refractive index of glass is 1.5, find ,the absolute index
of water.
(a) 1.33
(b) 2.33
(c) 0.33
(d) 0.44
Interpret

Here, the refractive index of glass with respect to water

ie, wll-g = 1.125 and absolute refractive index of glass llg

= 1.5.

We know that

c
v

Speed of light in vacuum


11

(b) 85 .3
(d) 65

(c) 60

= Speed of light in medium

Instance 6 Light is incident from air on oil at an angle of30. After


moving through oil-1, oil-2, and glass it enters water. If the refractive
indices of glass and water are 1.5 and 1.3, respectively, find the angle
which the ray makes with normal in water.
Air
Oil-1

Oil-2

Apparent Shift of an Object due to Refraction


Due to bending of light at the interface of two different
media, the image formation due to refraction creates an illusion
of shifting of the object position.
Consider an object 0 in medium. After refraction, the ray
at the interface bends. The bent ray when it falls on our eyes, is
perceived as corning from I.
For nearly normal incident rays, 81 and 82 will be very small.
Rarer

J
2 .6

(b) sin-

1
3.6

(d) sin- 1 (2 .6)

(a) sm -1 ( - 1
0

(c) sin-

Interpret

(-

J.lz

(_]__J
2.6

As we know
1-l sin i = (constant)
!l-air sin ~air)

=>

1-l-glass sin r (glass)


.

!l-air
=--Sin lair
1-l-glass
Again, llglass sin iglass = llwater sin r water
From Eqs. (i) and (ii)
sin 30 = 1.3 sin r
0

sm~glass)

1
2xl.3

1
2.6

... (i)
... (ii)

AB

AB

sm8 2 = - - - - - - - - - - - - - , - - - - - - Image distance from the refracting surface


1
2.6

smr=--=-, r=.sin-1 ( 0

tan 81 = sm 81 = -=-':-:-:-':__-:-:----:---.-=-:-----"':'--'---:----:-Object distance from the refracting surface


Similarly,

sin 81
sin8 2

_ 11 2 :::}
11
- 1 2 - 11
1

AB AB _
1
OB BI -

11 2
11
1

Chapter 23 Ray Optics


BI
OB

-=

v~

Apparent depth J..lz


= Real depth
J..L1

So, Shift = Real depth- Apparent depth = Real depth ( 1 -

~~ J

Case I
If ).ll < ).lz
Shift becomes negative , image distance > object distance,
ieJ image is farther from the refracting surface.

Total Internal Reflection

Whenever a ray of light goes from a denser medium to a rarer


medium it bends away from the normal. As angle of incidence
in denser medium increases, angle of refraction also increases in
rarer medium.
The angle of incidence in denser medium for whi.c h the
angle of refraction in rarer medium is 90 is called the critical
angle (C) .
sin C = ~Lrarer
sin 90 J..ldenser

Case II
If ).ll > ).lz.
Shift becomes positive, image distance < object distance,
ie image is closer to the refracting surface.
Case III
If ).lz = 1 or ).! 1 ~

Shift = Real depth ( 1 -

~J

Instance 9 A fish in an aquarium, approaches the left wall at


a rate of 3 ms-1, and observes a fly approaching it at 8 m.~- 1 . If the
refractive index of water is (4/ 3), find the actual velodty of the fly.

~X
).lX

(a) 3.75 ms- 1


(c) 0.75 ms- 1

(b) 2.75ms-1
(d) 4.75 ms- 1

Interpret

For the fish, appa.rent distance of the fly from the wall
of the aquarium is ).lX. If x is actual distance, then apparent

=~
J..ld

sinC=~

=>

J..ld

c = sin-1 (~:J

=>

).l

891

Now, if the angle of incidence in the rarer medium is greater


than the critical angle (C), then the ray instead of suffering
refraction is reflected back in the same (denser) medium.
This phenomenon is called total internal reflection. For
total internal reflection to take place following set of conditions
must be obeyed.
(i) The ray must travel from denser medium to rarer
medium.
(ii) The angle of incidence i must be greater than critical
angle C.

Instance 11 An isotropic point source (bulb) is placed at a depth h


below the water surface. A floating opaque disc is placed on the surface
of water, so that the bulb is not visible from the surface. What is the
minimum radius of the disc?
Take refractive index ofwater=).l.
Interpret

As shown in figure, light from bulb will not emerge out


1
of the water if at the edge of disc.

i>C

velocity will be d(J.!X)


dt

sin i >sin C

... (i)

(v.pp)fly = !l vfly

Now, the fish observes the velocity of the fly to be 8 ms-1


Therefore, apparent relative velocity = 8 ms- 1
1
Vfish + Cvapp)fly = 8 ms- ~ 3 + ).l vfly = 8
vf!y

= 5 x ~ = 3.75ms-1
4

Instance 10 A layer of oil 3 em thick is flowing on a layer of


coloured water 5 em thick. Refractive index of coloured water is 5/3
and the apparent depth of the two liquids appears to be 36/7 em. What
is the refractive index of oil?
(a) 1.4
(b) 2.4
(c) 3
(d) 2
Interpret

Apparent depth (AD) = .EL + 2_


5

J..lz

7
7

J..lz =- = 1.4
5

. c =-1
sm

So, Eq. (i) becomes


R

--===>~R2 +hz J..l

.2_= 36 -3= 15

or

J..l

-=--+7
5/3 J..lz

or

. .

smt=-,===
Jiz +hz
and

Ill ' 112

36

Now, if R is the radius of disc and h is the depth of bulb from it

or

h
R>--

~J..l-1

892

Chapter 23 Ray Optics

Refraction from a Spherical Surface


Spherical surfaces are of two types
(i) Convex
(ii) Concave

u'

Solving it we will get, v' = -35.4cm

1E'

v'

(4/ 3) = (3/ 2)
v'
( -40)

==>

A' B'' (Jl v')


.
Now usmg - - =-1, A' B'

2
();;,:I

llz_= llz

==>

=>

(Jl 2u')

A"B" = (3/ 2)(-35.4) => A"B" = 5.3cm


(5 .3)
(4/ 3)(-40)

The final images in all the above cases are shown in figure .
I

---p ------------------------

For both surfaces refraction formula is given by


~-~= Jllz-1
v u
R
11-1 2 is refractive index of second medium with respect to first.
If 1-1 1 and 1-lz are refractive indices of first and second medium
with respect to air, then,
llz _ J-11 = 1-lz 1-11
v
u
R

~'~

Lens

Lens is a transparent medium bounded by two curved


surfaces.
Lenses are of two types
1. Convex or convergent lens
2. Concave or divergent lens

1. Convex or Convergent Lens


The traqsparent medium bounded by two bulging surfaces
is called convex lens . It is ofthree types (as shown) .

instance 12 A linear object of length 4 em is placed at 30 em


from the plane surface of hemispherical glass of radius 10 em. The
hemispherical glass is surrounded by water. Find the final position and
size of the image:

(a) Double-convex
lens
(b) 4.3 em
(d) 2.3 em

(a) 5.3 em
(c) 5 em

;nterpret

For 1" surface 1-11 = -,J-1 2 =- ,u =-20cm,


3
2

(b) Plano-convex
lens

(c) Concavo-convex
lens

2. Concave or Divergent Lens


The transparent medium bounded by two hollow surfaces is
called concave lens. It is of three types (as shown).

R = +lOcm ,

and
8"

5.3cm
A"

(a) Double-concave (b) Plano-concave (c) Concavo-concave


lens
lens
lens

1 - - - - - v'

----~

1-lz _ J-11 = (J.lz -1-11)


v u
R
(3/ 2) - (4/ 3) = (3/ 2- 4/ 3)
==>
v
(-20)
10
==>
v = -30 em
Using
A' B' = ~ => A' B' = (4/ 3) (- 30)
AB
JlzU
( 4cm) (3/2) (-20)
==>
A'B' = 5.3 em
A'B' behaves as the object for plane surface
3
4
'
Jl 1 = ,Jlz=j and R=oo,u=-40
Using

Some Definitions Relating Lenses


Optical centre The optical centre is a point within or
outside the lens, at which incident rays refract without deviation
in its path.

Chapter 23 Ray Optics

8~~
l,i

Principal axis The straight line passing through the optical


centre of lens is called principal axis of lens.
1

Principal focus

Lens has two principal foci.

(i) First principal focus It is a point on the principal axis


of lens, the rays starting from which (convex lens) or appear to
converge at which (concave lens) become parallel to principal
axis after refraction.

Laws of Formation of Images by Lens

'

'
(i) The rays corning parallel to principal axis of lens pas~

through the focus after refraction.



(ii) The rays corning from the focus ofltilS go parallel tot~
principal a~is of lens after refraction.
':
(iii) The rays of light passing through optical centre go
straight after refraction without changing their path.

Lens Maker's Formula


...

.. ...
_...-:: F1

If R1 and R2 are the radii of curvature of first and seconq


refracting surfaces of a thin lens with optical centre C of foc~l
length/ and refractive index 1 ~2 then according to Lens Maker'$
formula

(ii) Second principal ,focus It is the point on the principal


axis at which the rays coming parallel to the principal axis
converge (convex lens) or appear to diverge (concave lens) after
refraction from the lens.

c
u-----1...._--v

..!..=(1~2-1)(_..!__ _ _..!__]
f

~
Both the foci of convex lens are real while that of concave
lens are virtual.

R2

..!..=(!l-l)(_..!__ _ _..!__J
f
R1 R2

where, 1 ~ 2 = ~ is refractive index of material of lens with'


respect to surrounding medium.
.
Thin lens formula is

1 1 1

-= - - ! v u

Focal length The distance between focus and optical


centre of lens is called focal length of lens.

Table 23.3

Rl

Formation of Image by a Convex Lens

1.

At infinity

At the principal focus (F2) or Real, inverted and extremely


in the focal plane
diminished

2.

Beyond 2F1

Between F2 and 2F2

Real, inverted and diminished

894

Chapter 23 Ray Optics

Real, inverted and of same size


as the object

4.

Between F1 and

Beyond 2F2

Real, inverted and magnified

2Fl

5.

At F 1

At infinity

Real, inverted and highly


magnified .

6.

Between F 1 and
optical centre

On the same side as the


object

Virtual, erect and magnified

Formation of Image by Concave Lens

Dividing Eq. (i) by (ii), we get

The image formed is always virtual, erect and diminished


and lies between the lens and F2 for all positions of the
object.

Instance 13 The focal length of convex lens is 10 em in air. Find its


focal length in water. (Given, llg. = 3/ 2 and llw = 4 / 3)
(a) 10 em
(b) 20 em
(c) 30 em
(d) 40 em
Interpret

1
- -=CI!g -1)(2__2_J
Rl

fair

and

(llg

ff'ater = llw -

Rz

XRl1 - Ri1 J

..

(i)

.. (ii)

fwater ( llg -1
"fair = llg f ll w -1

Substituting the values,


(3 / 2-1)
fwater = ( 312
)fair
- - -1
4/ 3
= 4 fair
= 4 X 10
= 40cm

Instance 14 An object is placed at a distance of 10 em to the left


on the axis of a convex lens L1 of fo cal length 20 em. A second convex
lens L2 offocal length 10 em is placed co-axially to the right of the lens
L 1 at a distance of 5 em from it. Find the position of the final image and
its magnification.
(a) 163. em on the right of the second lens, 3.33
3
.
(b) 163. em on the right of the second lens, 1.33
3
(c) 163. em on the right of the first lens, 1.33
3
(d) None of the above

Chapter 23 Ray Optics


Interpret

895

Important Features

Here, for 1" lens,

u1 = -10 em
f 1 = 20 em
1
1

vl

u1

1
1. Power of lens P = - -f(inm)

---=-

!1

=>

vl 20 10
v1 =-20cm

=>
L1

L2

, o;:;;

0 2 '!

';.:

\i
v
1'2

\ '!

,1,.\! Scm
1

= 2:1_ Vz = 20 _29_ = _ = 1.33


u1 u2 10 3x25 3

Height of image = _!__ = ~: ' ' ''


Height of object 00' u
Substituting v and u with proper sign, ) ,; .
.
If
-I
v

L,P;
n

The lateral, transverse or linear magnificauon produced by


a lens is defined by
Height of image I
m=
=Height of object 0
A real image II' of an object 00' formed by a convex lens is
shown in figure.

Thus,

i=1

Magnification of Lens

or

i=l

Magnification of combination
50

00' 0
-u
I
v
-=m=0
u
v
m=-

p =Pl +Pz + ... =

ie, final image is at a distance o; 163_ em on the right of the


second lens.
,
3
T)le magnification of the image is given by;

'.

L.-1

Power of combination

M = m1 xm2 x ... =11m

- + - = - => v2 =-=16-cm
v2 25 10
3
3

~ala2 .

This is Newton's formula.


3 . If two or more lenses are placed in contact, then
equivalent focal length of the combination.

For 2nd lens, - - - = Vz u2 fz


Here, u2 = -(20 + S),J2 = 10cm
1

1 1 1
-=-+-+ .. =
f !1 fz

ie, the image is virtual and hence lies on the same side of the
object. This will behave as an object for the second lens.
1

100
j(incm)
Power of convex lens is positive and of concave lens is
negative .
2. If distance of an object from first focus of lens is a 1 and
distance of image from second focus is a 2 , then its focal
length.
P=

=>

1
1
1
-=---

i=l

4. If two lenses offocallengthsf1 andf2 are separated by a


distance x, then its equivalent focal length
1

-=-+ - - F !1 fz fdz
Power of combination,
P = P1 + Pz- x P1P2
Total magnification remains unchanged ie,
m=m 1 xm 2
5. If a lens is made of a number
of layers of different refractive
indices, then number of images
of an object formed by the lens
is equal to number of different
media.
6. Cutting of a lens
(i) If a symmetrical convex
lens of focal length f is
cut into two parts along
its optic axis, then focal length of each part (a
plano convex lens) is 2f. However, if the two parts
are joined as shown in figure, the focal length of
combination is again f.
2(,

(a)

(b)

2f

(c)

(d)

896

Chapter 23 Ray Optics


(ii) If a symmetrical convex lens of focal length f is cut
into two parts along the principal axis, then focal
length of each part remains changed at f . If these two
parts are joined with curved ends on one side focal
length of the combination is f_ . But on joining two
2

parts in opposite sense the net focal length becomes


(or net power = 0).

7. Silvering of a lens
(i) Let a plano-convex lens is having a curved surface of
radius of curvature R and has refractive index J.l if
its plane surface is silvered, it behaves as a concave
mirror of focal length.

R
=- 2(~-t- 1)

(ii) If the curved surface of plano-convex lens is silvered

then it behaves as a concave mirror of focal length.

R
=- 2!l

(iii) If one surface of a symmetrical double convex lens


(R1 = R2 = R) is silvered, then the lens behaves as a
concave mirror of focal length

f=
(a)

Table 23.4

8 . The tabular difference between lens and mirror is given


(b)

(c)

(d)

in table.

Difference between Lens and Mirror

1.

Convex lens

+ ve

+ ve

converging

2.

Concave mirror

- ve

+ ve

converging

3.

Concave lens

- ve

-ve

diverging

4.

Convex mirror

+ ve

-ve

diverging

2(2J.l-l)

897

Chapter 23 Ray Optics


Instance 15 A convergent lens of 6 D is combined with
a diverging lens of -2 D. Find the power and focal length of the
combination.
(a) 26 em
(b) 20 em
(d) 25 em
(c) 30 em
Interpret Here, P 1 = 6 D, P2 = -2 D
Using the formula, P = P 1 + P2 = 6- 2

Instance 16 A convex lens of 10 em focollength is combined with a


concave lens of 6 em focollength. Find the focollength of the combination.
(a) -15 em
(b) 15 em
(c) 10 em
(d) -10 em
Interpret Here,f1 = 10 cm,f2

.!. = ..!._+..!._ = _.!__.!_ = _ _.!_

Use the formula

= 4D

= -6 em, F =?

!1 ! 2

' F

f = liP= 1/4 m = 25 em.

10

<Jl

15

F = -15 em

Is the ratio of frequencies o_f ultraviolet rays and ......,,a""'


Can convergent
Why

!I)

Refraction Through a Prism

or

A prism is a homogeneous,
p
transparent medium bounded
by two plane surfaces inclined
A
at an angle A with each other.
These surfaces are called as
refracting surfaces and the
angle between them is called
angle ofprismA.
Figure
shows
the
0
R
refraction of monochromatic
light through a prism. Here' i and e represent the angle of
incidence and angle of emergence respectively, r 1 and r2 are two
angles of refraction. If J.l is the refractive index of the material of
the prism, then
sini sine
J.l=--=-sinrl sinr2

A+om
2

!=--

[A+om)

sm - -

smt

J.!=-.- or J.l=
smr

For thin prism,

om" {ll

l)A.

Instance 17 The angle of minimum deviation for a glass prism


with J.l =
equals the refracting angle of the prism. What is the angle
of the prism?

.J3

Interpret
Using,

ut

we have

Minimum Deviation
or

A
. A
2 Slll - COS 2
2

sin(1)

sin(1)

.J3

A
- = 30 or A = 60
2
.
L---~---:i-="-e----.i1

o= om;= (i + 0 -A
1

sinA

.J3=--=

COS-=-

A
r=-

Further at

sin -

The angle between the incident ray and the emergent ray
is lmown as the angle of deviation o. For refraction through a
prism it is found that
i + e =A + o and r 1 + r2 = A

It is found that the angle


of deviation o varies with
the angle of incidence i of
the ray incident on the first
refracting face of the prism.
The variation is shown in Om
figure and for one angle of
incidence it has a minimum
value omm. At this value
i=e
It therefore, follows that

r1

= r2

~~

.Dispersion

Dispersion of light is the phenomenon of splitting of white


light into its constituent wavelengths on passing through a
dispersive medium, eg, prism. Cause of dispersion is the variation
of refractive index of prism with wavelengJ:h. As A.v < A.R, hence,
J.lv > llR and consequently Ov > Ow

898

Chapter 23 Ray Optics


Instance 18 Find the dispersion produced by a thin prism of 18
having refracting index for red light = 1.56 and refractive index for
violet light = 1.68.
(a) 2.16
(b) 1.16(c) 3.16
(d) 2.10
Interpret

We know that dispersion produced by a thin prism

e = C!-lv -11R)A
Here, 1-lv

Angular Dispersion It is the angular separation between


the fwo extreme rays.
Angular dispersion 8 = 8v - 8R
= C11v - 11R )A
Dispersive Power The dispersive power of a prism
material is measured by the ratio of angular dispersion to the
mean deviation suffered by light beam.
:. Dispersive power
ro- Ov -oR _ 1-lv -1-lR

--0--~,

where 11 is the mean value of refractive index of prism.


The dispersive power of a prism depends only on its material
and is independent of angle of prism, angle of incidence or size
of the prism.
Dispersive power is a unitless and dimensionless term.
Dispersive power of a flint glass prism is more than that of
a crown glass.

Dispersion without Deviation (Direct Vision Prism)


1. To produce dispersion witho~t mean deviation we use a
combination of two prisms of different materials such that

= 1.68,~-tR = 1.56 and A

= 18

e.=(1.68 -1.56) x 18 = 2.16


Instance 19

Calculate the dispersive power for crown glass from

the given data


1-lv

= 1.523

(a) 0.01639
(c) 0.05639

and 1-LR = 1.5145


(b) 1.05639
(d) 2.05639

Interpret

Here, 1-lv = 1.523 and 1-lR = 1.14S.


. . d
l.S23+1.S145
Mean re fr actlve m ex, 11 =
l.S187S
2

Dispersive power ro is given by,


ro = !lv -llR = l.S23 -l.S14S
(!l-1)
(l.S187S-1)

_
0 01639

Instance 20 A prism of crown glass with refracting angle of so


and mean refractive index = 1.51 is combined with a fiint glass prism
of refractive index = 1.6S to produce no deviation. Find the angle of
fiint glass .
(a) 3.92
(b) 4.68
(c) 5.32
(d) 7.28
Interpret

Let A' be the angle of flint glass prism.

Here, A

= so and J-l = 1.51 for crown glass prism.

8 = (!l-1)A= (l.S1-1)xS = 2.SS 0

Deviation produced by flint glass

o' = (!!' - 1)A' = (1.6S -1)A' = 0.6SA'

For no deviation, 8' = 8 or 0.65A'

2.55

A'= 2.S5 = 3.92o


0.65

> Optical Instruments

A'= -(l-l.-1JA
!-l-1

2. Net dispersion caused


= C11v- 11R) A + C11' v - 11' R)A'
= (11- 1)A (m - m') = 8 (m - m')

Deviation without Dispersion (Achromatic Prism)


1. To produce deviation

without dispersion
we use a combination
of two prisms of
different materiaPs
such that
~A'=- [!-lv -1-lR] A
[!-l~ -!-l~]

2. Resultant

~eviation produced =O[1-:]

Optical instrument is a device which is made from proper


combination of mirrors, prisms and lenses. The principle of
working of optical instruments depends on laws of reflection and
refraction of light.

Microscope
It is an optical instrument which forms a magnified image
of a small nearby object and thus, increases the visual angle
subtended by the image at the eye so that the object is seen to be
bigger and distinct.
)

..

(i) Simple microscope A simple microscope is a convex


lens of short focal length which is fixed in a frame provided with
',
handle .
. cj

Chapter 23 Ray Optics


Magnification of simple microscope
(a) When final image is formed at least distance of
distinct vision,

899

Object

M=1+-

(b) For relaxed eye, M = D

f
where D

least distance of distinct vision.

Magnification of astronomical telescope

Figure shows a simplified


version of a compound microscope . It consists of two converging
lenses arranged coaxially. The one facing the object is called
objective and the one close to eye is called eye piece. The
objective has a smaller aperture and smaller focal length than
those of the eye piece.
(ii) Compound microscope

(a) For relaxed eye, M = _ fo


~
fe
In this position, length of telescope
L~ =fo +fe

(b) When final image is formed at least distance of


distinct vision

M = _ f o ( 1 + fe
D
fe
D
Length of telescope
LD =fo+ue
f o = focal length of objective lens
f e = focal length of eye piece

Magnification of compound microscope


(a) For relaxed eye

M~ = _ vo (E-J
Uo

fe

In this position, length of microscope


L~ = Vo

(ii) Terrestrial telescope In an astronomical telescope ,


the final image is inverted with respect to the object. To remove
this difficulty, a convex lens of focallengthf is included between
the objective and the eye-piece in such a way that the focal plane
of the objective is a distance 2f away from this lens.

+ fe

(b) When final image is formed at least distance of distinct

1~~~~~
A

VlSlOn.

MD

=~(1+ DJ
fe
Uo

Length of microscope,
L0 =v0 +Ue
v0 = distance of first image from object lens.
U 0 = distance of object from objective lens.
f. = focal length of eye piece.

Telescope
Telescope is an optical instrument which increases, the
visual angle at the eye by forming the image of a distant object
at the least distance of distinct vision, so that the object is seen
distinct and bigger.
(i) Astronomical telescope It consists of two converging
lenses placed coaxially. The one facing the distant object is called
the objective and has a large aperture and large focal length.
The other is called the eye-piece, as the eye is placed closed to
it. The eye-piece tube can slide within the objective tube, so that
the separation between the objective and the eye-piece may be
varied.

'

~-r0 ~2r--- e r-~

Magnification of terrestrial telescope


(a) For relaxed eye,

M~ =

In this position, length of telescope


L~ = f o + 4f + f e
(b) When final image is formed at least distance of distinct
vision,

M = fo (1+ fe
D
fe
D.
Length of telescope ,
n

LD = fo + 4 f + de
f o = focal length of objective lens
f e = focal length of eye peice
Galilean telescope A simple model of Galilean telescope
is shown in figure. A convergent lens is used as the objective and
a divergent lens as the eye-piece.

900

Chapter 23

* Ray Optics

~--------

fo _ _ _ _ _ ____..,

Resolving Power of a Microscope


Resolving p;Jwer of e. microscope is defined as the reciprocal
of the least separation between two close objects, so that they
appear just separated, when seen through the rnicroscope.
The least separation between two objects, so that they
appear just separated is given by

1
A

d=--"'2J.1sin8
where fl is che refractive index of the medium between the
objective of the microscope and the object. This distance is called
limit of resoh~tion of the microscope.

Magnification of Galilean telescope


(a) For relaxed eye,

M~ =

fo

fe

In this position, length of telescope


L, = fo- fe
(b) When final image is formed at least distance of distinct
vision M = fa
'
D
fe

Resolving power of a micro5mpe = .!_ = 2J.1 sine


d
lc
8 = half angle of the cone of light from the point object,
fl sin 8 = numerical aperture

(1 - fe J
D

....de._____ _____ _

Length of telescope

Lv = fo -u,

Instance 21 An object is seen through a simple microscope offocal


length 20 em. Find the angular magnification produced if the image is
formed at 30 em from the lens.
(a) 2.08
(b) 2.05
(c) 3.08

Interpret
and

(d) 1.5
1.

Resolving power of microscope increases with increase in the value


of the refractive index of the meaium between objective and object
that's why oil immersion objective microscopes are used to achieve
high resolving power.
2. The resolving power of microscope increases, with decrease in the
value of the wavelength of the light used to illuminate the object,
so microscopes using ult~aviolet light for illu(Tlinating the objects
are used to achieve high resolving power. These are called ultra
microscopes.
Higher resolving power is obtained in electron microscope.

Given,f = + 20 em
v = - 30 em ,

Using thefonnula,
1
-30

.!._.!.=.!.
v

wehave,

1
20
u0 = 12 em

-U 0

'{<The >angular magnification, M

=.!!_ = 25 =2.08
uo

12

Instance 22 A galilean telescope is 27 em long when focussed to


form an image at infinity. If the objective has a focal length of 30 em,
what is the focal length of the eye piece?
(a) 3 em
(b) -3 em
(c) 2 em
(d) -2 em
Given,fo = + 30 em.
Objective
Eye-piece
Length of telescope is given
27cm.
Therefore, u, = + 3 em.
For the final image at infinity,
the intermediate image should lie
at first focus of eye piece of the I-- 27 em --l~
Galilean telescope.
1---~- 30 em
I
fe=-3cm

Interpret

> Resolving Power of Telescope


Resolving power of telescope is defined as the reciprocal
of the smallest angular separation between two distant objects,
so that they appear just separated, when seen through the
telescope.
The smallest angular separation between two objects, so
that they appear just separated is found to be
de= 1. 2 21c
D

where D is the diameter of objective


Resolving power of telescope

1.22/c

lntext Questions 23.3


en does a ray fucid~nf on _a prism deviate away from the base?
"ngs) observed sometimes round the sun or moon?
f gla.~s for lights ?f yellow, green and red colours are f!y. llg and flr respectively. Rearrange 'thes~
fv~l~7 1 : _;
e position a object relative to a biconvex lens so that it behaves like magnifying lens?
erted, ~ill it serve as a microscope?

of

901

Chapter 23 Ray Optics

Chapter Compendium
10. Refraction at Sphe,rical Surfaces,

are o\:1eyed at every reflectin,g surface.

(i) For a spheric1il surface, f.Lz. ..., f.LI =

Hz - f.L1

ofsy:mmetrical spherical surfaces, are


two
(i) convex, and (ii) concave.
which the reflection takes place at the bulged
a convex mirror and the mirror in which
place at the depressed surface is called a

(ii) Magnification, m

= 111::_
flzU

(iii) When the object is

relation can be obtainted


relation becomes

vv<<.:muL)(.UI

_ f.L2 +~t1 = !11 -J.Lz


u v
R
11. Thin Lens

.Ll ~'"wouL

rays, the refracted ray and the normal


surface separating the two media-all lies in one

law For any two media the ratio of the sine of the
of incidence to the sine of the angle of refraction
constant for a light beam of a particular frequency, ie

12. Thin lens fonnula .,--- =-

v u f

13. Linear magnification of a lens


14. Power ofLens

= (1/J) where f is the focal


the lens in meters with proper sign. ..
(ii) Power of combination of lenses.P ':"
where P1, P2 , P3 , are powers
(iii) Magnification produced by equiValent lens.uf =
15. Prism
(i) Refracting angle
(i) Power of a lens

o{co

T
t

L--..-~...,...--~~,,

(ii) A

'

C is the critical angle

+ o=i + e

m~

;5 ,m2

902

"

Chapter 23 Ray Optics

s .

l
l

"l

'

Illustrative
Example 1 A beaker contammg liquid is placed on a table
underneath a microscope which can be moved along a vertical scale.
The microscope is focussed, through the liquid, on a mark on the
table and the reading on the scale is a. It is next focussed on the upper
surface of the liquid and the reading is b. More liquid is added and the
observations are repeated, the corresponding readings being c and d.
The refractive index of the liquid is
(a)

d-b
d-c-b+a

(b)

b-d
d -c-b+a

(c)

d-c-b+a
d-b

(d)

d-b
d+c-b-a

/.12

Using Eqs.(iii) and (iv), we obtain


sini = 112
cosi 111

Solution

The real depth = (Refractive index) apparent depth


::::>In first case, the real depth h 1 = ).!(b- a).
Similarly, in the second case, the real depth h2 = J.!(d- c)
Since, h2 ~ h1 the difference of the depths = h 2 - h 1
= J.!(d- c- b + a)
Since, the liquid is added in second case, h2 - h1 = d - b
d-b
ll= d-c-b+a

Example 2 A ray of light from a denser medium strikes a rarer


medium at angle of incidence i. The reflected and refracted rays make
an angle of90 with each other. The angles of reflection and refraction
are r and r', respectively. The critical angle is

tani = 1-lz
Ill
Since, at the time of total internal reflection,
sinec

= 1-lz,
Ill

... (v)

using Eq.(v) we obtain ec = sin-1 (tanr) .

Example 3 A given ray of light suffers minimum deviation in an


equilateral prism R Additional prism Q and R of identical shape and of
the same material as P are now added as shown in the figure. The ray
will now suffer

(a) greater deviations


(b) no deviation
(c) same deviation as before (d) total internal reflection

Solution

No deviations occur on interfaces 2 and 3 as there is no


change in medium. However, deviation at interface 4 is same as
it was on interface 2 with only prism P.

(a) sin- 1 (tan r)


(b) sin-1 (cot i)
(c) tan- 1 (sin r)
(d) tan- 1 (sin i)

Solution

Example 4 The sun (diameter D) subtends an angle of e rad


at the pole of a concave mirror of focal length f The diameter of the
image of the sun formed by the mirror is
(a) f6
(b) f26!D
(c) 2f6
(d) De

Applying Snell's law for refraction,


sini = 11 2
sinr' 111

... (i)

From the given condition, r + r' = 90


sin r' =cos r
Solution of Eqs. (i) and (ii) yields, sin i
cosr
According to the Law for refraction

=r

... (ii)

= 112
111

.. . (iii)

... (iv)

Solution

Since, the sun is at very large distance, u = oo

904

Chapter 23 Ray Optics

1 1 1

Example 7 When an object is at distances of u 1 and u 2 from the


poles of a concave mirror, images of the same size are formed. The focal
length of the mirror is

-+-=-

v f
==>
v=f
If the diameter of the image be d,
00

(r)

lu1 +uzl
lu1 -uzl
lul ;uzl

(d)

lu1;u2 1

(a)
(b)

T
d

2
.J_

d/2
--=a:::> d = (2a)v
v
:::>
d=(2a)v
Putting, 2a =6 and v = f, we obtain d = f6
A ray enters a glass sphere of refractive index fl = ../3 at
an angle of incidence 60 a ray is reflected and refracted at the farther
surface of the sphere. The angle between the reflected and refracted rays
at this surface is
(b) 60
(a) 50
(d) 40
(c) 90

Solution

One image will be real and the other will be virtual.


___Since they are of the same size, one will have magnification m
and the other -m.
1

Example 5

Solution

-+-=-

ul u1m f
2_(1 + ]:_) =_!_

or

_ u1

and

... (i)

Uz

u2m

----=-

Refraction at P

2_(1-]:_)=.!.

or

u2

... (ii)

From Eqs. (i) and (ii), we get

ul +uz = 2
f
f

or
sin 60 = ../3 .
sinr1
sinr1 = (1/2) ==> r1 = 30
.l

Since

r2
r2

ul +uz
2

Example 8 A ray of light passes through four transparent media


with refractive indices J..lJJ J..lz, J..t3 and J..l4 as shown in the adjacent figure .
The surfaces of all media are parallel. If the emergent ray CD is parallel
to the incident ray AB, we must have

= r;
= 30

sinr2
1
Refraction at Q = - -sini2 - ../3
Putting r 2 = 30, we obtain i2 = 60
Reflection at Q, r; = r2 = 30

Example 6 A soldier directs a laser beam on an enemy by reflecting


the beam from a mirror. If the mirror is rotated by an angle 8, by what
angle will reflected beam rotate?
(a) 8/2
(b) 8
(c) 28
(d) None of these

(a) J..l1
(c) J..l4

Solution
i1 =

= J..l3
= J..l1

(b) J..l2
(d) J..l2

= J..l4
= J..l3

Considering Snell's law, J..l sin 8 = constant, and

4 (given).

Let MPM2 be the initial position of the mirror.


The mirror is rotated through an angle 8 to the position
M't OM2.PO is the incident light. OQ on the initial reflected
ray and OQ' is the reflected ray after rotating the mirror by
angle 8. If i = initial incidence angle, then L.POQ = 2i and
L.POQ' = L.PON' +L.N'OQ = 2i- 26.

Solution

Q'

sin i1 = flz sm 12
sin i2 111 ' sin i3

:. The reflecte~ beam rotates through an angle 28.

Since,

= 11 3 sini3 = 11 4
112 '-sin i4 113
sini1 = 11 4
sin i4 111
i1 = i4 , therefore J..t4

= J..t 1

Chapter 23 Ray Optics


Example 9 Light is incident normally on face AB of a prisTTJ. as
shown in the figure. A liquid of refractive index J.1 is placed on face AC
of the prism. The prism is made of glass of refractive index 3/2.
Tile limits of fl for which total internal reflection takes place on
faceAC is

905

.
1 1
1
1
Nowusmg, - = - + - + - wehave
F f 1 f z f3
1 1 1 1
-=-+-+-

F f

2
3f

f'

4
3f

=---=-

F= 3f
4

Example U

A concave lens jQrms the image of an object such


that the distance berween the object. and image is 10 em ana the
magnification produced is 1/4. The focal (ength of the lens will be
(a) 8.6 em
(b) 6.2 em

3J3

(b) ll < - -

(a) !J.> -

Solution

.)3

(d)

Solution

(c) 10 em

So let

!J.<2

Critical angle between glass and liquid face is


sine
c

= 312 = -~
ll

... (i)

21J.

Angle of incidence at face AC is 60


ie,
i = 60
For total internal reflection to take place

i > ec
sin i >sin ec

or
or

. 600 >-3
sm
2}l

or

.)3 3
->2
2}l

or

}l> .)3

-------------------------------------------_-:_-_:::.-_-_-_:::::.-_:::.-_-_-_:::.-_-:_-_:

(b)

Let R be the radius of curvature of each surface. Then

J.

For the water lens


; . =(i- 1)( -*+*) =M

f = - 4.4 ern

0
3x

4x

Length of the tube is L =

V0

.!.. - .!.. = .!..


Vo

Uo

1
22.5

U0

+f

= L- F.=

~5

we have

fo

1
1.5

----=-

1 1
1) ( "R+"R

or

Now applying

(d) 3f

1
57=CL

f=-~

V0

f_

~olution

or

Solution

(c) 4f

u=-4xandv=-x
Then 3x = 10 ern
10
or
x =-ern
3
40
10
u=--cmand v=--cm
3
3
. . . 1 -3 3
Su b stltutmg m - = - + ! 10 40

Example 12 A microscope has an objective of focal length 1.5 em


and eye piece of focal length 2.5 em. If the distance between objective
and eye-piece is 25 em, what is the approximate value of magnification

produced for relaxed eye?


(a) 75
(b) 110
(c) 140
(d) 25

Example 10 Two identical glass (J.18 =3!2)


equiconvex lenses of focal length fare kept in contact.
.The space between the two lenses is filled with water
Cflw = 4/3). The focal length of the combination is
(a)

(d) 4.4 em
Concave lens fom1S the virtual image of a real object.

luolz1.6 ern
IMI=vox!!_

f.
=(22.5) (~)uo

-7l ;.

=-

3~

\ 1.6

z140.

2.5

- 2.5

= 22.5 ern

Chapter Practice
Exercise I
Photometry
1. What is the ratio of luminous intensity ot two sources,
which produce shadows of equal intensities at ci1stance
25 em and 50 em from the photometer screen?
(a) 1 : 4
(b) 4 : 1
(c) 1 : 2
(d) 2 : 1
2. The time required for making a print a distance of 0.25 m
from a 60 W lamp is 5 s. If the distance is increased to
40 em, the time required in second to make a similar print
is
(a) 3.1
(b) 8
(d) 16
(c) 12.8
3. A source is at 4m height above the centre of a circular table
of a circular table of radius 3m. The ratio of illuminance
at 0 and P will be
, S

(a) 10%
(b) 20%
(c) 27%
(d) 36%
7. In a photometer, two sources of light when placed at
30 em and 50 em respectively produce shadows of equal
intensities. Their candle powers are in the ratio of
(a)

8.

9.

4m

PF---3-m_...Jo

10.

64
(a) 125

(b) 125
64

(c) 1

(d) 16

25
4. A lamp of 250 candela power is hanging at a distance of
6 m from a wall. The illuminance at a point on the wall at
a minimum distance from lamp will be
(a) 9.64lux
(b) 4.69 lux
(c) 6.94lux
(d) None ofthese
5. An electric bulb illuminates a plane surface . The intensity
of illumination on the surface at a point 2 m away from
the bulb 5 x 10- 4 phot (lumen cm-2 ) . The line joining the
bulb to the point makes an angle of 60 with the normal
to the surface. The intensity of the bulb in candela (candle
power) is
'
(a) 40 x 10- 4
(b) 40
(d) 20
(c) 40vG
6. A lamp is hanging at a height of 40 em from the centre
of the table. If its height is increased by 10 em, the
illuminance of the lamp will decrease by

11.

_.2._
25
3

(b) 16

25
5
(c) (d)
5
3
A book can be read if it is placed at a distance of 50 em
from a source of 1 cd. At what distance should the book
placed if the source is of 16 cd?
(a) 8 m
(b) 4 m
(c) 2m
(d) 1 m
In a grease spot photometer, light from a lamp with dirty
chimney is exactly balanced by a point source distance
10 em from the grease spot. On dearing the dirty chimney,
the point source is moved 2 em to obtain a balance
again. Then the percentage of light absorbed by the dirty
chimney is nearly
(a) 64%
(b) 36%
(c) 44%
(d) 56%
Two point sources A and B of luminous intensities 1 cd
and 16 cd respectively are placed 10.0 em apart. A grease
spot screen is placed between the two sources. For the
grease spot to become indistinguishable from both the
sides, it should be placed at
(a) 80 em from 16 cd lamp and 20 em from 1 cd
(b) 20 em from the 16 cd and 80 em from 1 cd
400
100
(c) - - em from 16 cd and em from 1 cd
3
3
400
100
(d)
em from 16 cd and
em from 1 cd
3
3
A point source of light moves in a straight line parallel
to a plane table. Consider a small portion of the table
directly below the line of mowment of the source. The
illuminance at this portion varies w1tn this d1stance r from
the source as
~1

(a)
(c)

oc

-;z1

(b)

oc

(d)

=-;A

12. As the wavelength is increased from violet to red, the


luminosity

Chapter 23 Ray Optics


(a)
(b)
(c)
(d)

19. A fish is a little away below the surface of a lake. If the

continuously increases
continuously decreases
increases then decreases
decreases then increases

critical angle is 49, then the fish could see things above
water surface within an angular range of 8 where
Air
Water

Reflection of Light
13. A dentist has a small mirror of focal length 16 mm. He

views the cavity in the tooth of a patient by holding


the mirror at a distance of 8 mm from the cavity. The
magnification is
(b) 1.5
(a) 1
(d) 3
(c) 2
14. Given width of aperture = 3 mm and 1c = 500 nm. For
what distance ray optics is good approximation?
(a) 18m
(b) 18 mm
(c) 18A
(d) 18 light years
15. The separation between the screen and a plane mirror
is 2r. An isotopic point source of light is placed exactly
mid way between the mirror and the screen. Assume that
mirror reflects 100% of incident light. Then the ratio of
illuminance on the screen with and without the mirror is
(a) 10 : 1
(b) 2 : 1
(d) 9 : 1
(c) 10 : 9
16. An object is placed asymmetrically between two plane
mirrors inclined at an angle of 72. The number of images
formed is
(a) 5
(b) 4
(c) 2
(d) infinite
17. From a spherical mirror, the graph of 1/v versus 1/u is
1

v
(b)

(a)

Ti

v
(d)

(c)

Ti

by

(a) 8=49
"(c) 8=24_!_ 0
4

49

(b)

e = 98

(d) 8=90

20. A car is fitted with a convex mirror of focal length 20 em.


A second car 2 m broad and 1.6 m height is 6 em away

from the first car. The position of the second car as seen
in the mirror or the first car is
(a) 19.35 em
(b) 17.45 em
(c) 21.48 em
(d) 15.49 em
21. A convex mirror forms an image one-fourth the size of the

object. If object is at a distance of 0.5 m from the mirror,


the focal length of mirror is
(a) 0.17 m
(b) -1.5 m
(c) 0.4 m
(d) - 0.4 m
22. A person 6 feet in length can see his full size erect image
in a mirror 2 feet in height. This mirror has to be
(b) plane or concave
(a) plane or convex
(c) necessarily convex
(d) necessarily concave
23. A point object is placed at a distance of 30 em from a convex
mirror of a focal length 30 em. The image will form at
(a) infinity
(b) pole
(c) 15 em behind the mirror
(d) no image will be formed
24. An object is placed at a distance of 10 em from a concave

18. For a convex mirror, the variation of u versus v is given

mirror of radius of curvature 0.6 m. WJ:tich ofthe following


statements is incorrect?
(a) The image is formed at a distance for 15 em from the
mirror.
(b) The image formed is real.
(c) The image is 0.5 times the size of the object.
(d) The image is 1.5 times the size of the object.
25. With a concave mirror, an object is placed at a distance x 1
from the principal focus, on the principal axis. The image
is formed at a distance x 2 from the principal focus. The
focal length of the mirror is _

(b)

(a)

(b)

u
v

(c)

(d)

907

(d)

xl +xz

Jxx
1 2

26. A man has a concave shaving mirror of focal length 0.2 m.


How far should the mirror be held from his face in order
to give an image of two fold magnification?
(a) 0.1 m
(b) 0.2 m
(c) 0.3 m
(d) 0.4 m

908

Chapter 23 Ray Optics

2 7. To focal length of a concave mirror is 12 em. Where should


an object of length 4 em be placed so that an image 1 em
. long is formed?
(b) 3 em
(a) 48 em
(d) 15 em
(c) 60 em

36. Two plane mirrors are inclined to each other at an angle 8.


A ray of light is reflected first at one mirror and then at
the other. The total deviation of the ray is

28. The focal length of a concave mirror is 20 em. Where an


object must be placed to form an image magnified two

37. A plane mirror is approaching you at 10 cms-1 . Your image


shall approach you will a speed of
(a) + 10 cms-1
(b) - 10 cms- 1
(c) + 20 cms-1
(d) - 20 cms- 1

times when the image is real?


(a) 30 em from the mirror
(b) 10 em from the mirror
(c) 20 em from the mirror
(d) 15 em from the mirror

31.

32.

33.

34.

the object is
(b) 36 em
(a) 18 em
(d) infinite
(c) 48 em
Sun subtends an angle of 0 .5 at the centre of curvature
of a concave mirror of radius of curvature 15 m. The
diameter of the image of the sun formed by the mirror is
(a) 8 .55 em
(b) 7.55 em
(c) 6.55 em
(d) 5.55 em
A convex mirror and a concave mirror has radii of
curvature 10 em each are placed 15 em apart facing each
other. An object is placed midway between them. If the
reflection first takes place in the concave mirror and then
in convex mirror. the position of the final image is
(a) on the pole of the convex mirror
(b) on the pole of the concave mirror
(c) at a distance of 10 em from convex mirror
(d) at a distance of 5 em from concave mirror
An object 5 em tall is placed 1 m from a concave spherical
mirror which has a radius of curvature of 20 em. The size
of the image is
(a) 0.11 em
(b) 0.50 em
(c) 0.55 em
(d) 0.60 em
A convex mirror of radius of curvature 1.6 m has an object
placed at a distance of 1 m from it. The image is formed at
a distance of
(a) 8/13 min front of the mirror
(b) 8/13 m behind the mirror
(c) 4/9 min front of the mirror
(d) 4/9 m behind the mirror
A short linear object of length b lies along the axis of a
concave mirror of focal length f at a distance u from the
pole of the mirror. The size of the image is equal to

(a) b( u;ffz
b(u;f)
(c)

(b)

(d)

b(_L)1/2
u-f
b(f~u)

35. When an object is kept at a distance of 30 em from a


concave mirror, the image is formed at a distance of
10 em. If the object is moved with a speed of 9 ms- 1 , the
speed with which images moves is
(a) 0.1 ms-1
(b) 1 ms- 1
1
(c) 3 ms(d) 9 ms- 1

(b) 240- 28
(d) 180-8

38. A candle is placed before a thick plane mirror. When

29. A spherical mirror forms diminished virtual image of


magnification 1/3. Focal length is 18 em. The distance of

30.

(a) 28
(c) 360-28

looked obliquely in the mirror, a number of images are


seen from the surfaces of the plane mirror. Then
(a) first image is brightest
(b) second image is brightest
(c) third image is brightest
(d) all images beyond second are brighter
39. An object is approaching a plane mirror at 10 cms- 1 A
stationary observer sees the image. At what speed will the

image approach the stationary observer?


(a) 10 cms-1
(b) 5 cms-1
(c) 20 cms-1
(d) 15 cms-1
40. A small object is placed 10 em in front of a plane mirror.
If you stand behind the object, 30 em from the mirror and
look at its image, for what distance must you focus your
eyes?
(a) 20 em
(b) 60 em
(c) 80 em
(d) 40 em
41. When a convergent beam of light is incident on a plane
mirror, the image formed is
(a) upright and real
(b) upright and virtual
(c) inverted and virtual
(d) inverted and real
42. It is necessary to illuminate the bottom of a well by
reflected solar beam when the light is incident at an
angle of a = 40 to the vertical. At what angle p to the
horizontal should a plane mirror be placed?
(a) 70
(b) 20
(c) 50
(d) 40
43. The sun (diameter d) subtends an angle 8 radian at the
pole of a concave mirror of focal length f . The diameter of
the image of sun formed by mirror is
(a) 8f

(b)

~f
2

(c) 28
(d)

~f

n
44. A plane mirror reflects a pencil of light to form a real
image. Then the pencil of light incident on the mirror is
(a) parallel
(b) convergent
(c) divergent
(d) Any of these
45. A spherical mirror forms an image of magnification 3. The
object distance, if focal length of mirror is 24 em, may be
(a) 32 em, 24 em
(b) 32 em, 16 em
W32~o~

~16~~~

Chapter 23 Ray Optics

Refraction of Light
46. How will the image formed by a convex lens be
affected, if the central portion of the lens is wrapped
in blank paper, as shown in the figure .
(a) No image will be formed
(b) Full image will be formed but is less bright
(c) Full image will be formed but without the central
portion
(d) 1\vo images will be formed, one due to each exposed
ha~

47. The distance v of the real image formed by a convex lens is


measured for various object distance u. A graph is plotted
between v and u . Which one of the following graphs is
correct?
v

50. A layered lens as shown in figure is made of two types


of transparent materials indicated by different shades.
A point object is placed on its axis. The object will form

(a) 1 image
(b) 2 images
(c) 3 images
(d) 9 images
51. As shown in figure position of an images I of an object 0
formed by lens. This is possible if

oLJ

(a)

(b)

u
v

(c)

(d)

'~

(a) a convex lens is placed to the left of 0 .


(b) a concave lens is placed to the left of 0
(c) a convex lens is placed between 0 and I
(d) a concave lens is placed to the right of I
52. The relation between n1 and n 2 if the behaviour of light
ray is as shown in the figure.

48. If the space between the lenses in the lens combination


shows were filled with water, what would happen to the
focal length and power of the lens combination?

~ens

(a) n 2 > n 1
(c) n 1 > n 2

I
/

(b) n 1 >> n 2
(d) n 1 = n 2

53. A convex lens A of focal length 20 em and a concave lens


B of focal length 56 em are kept along the same axis with
the distance d between them. If a parallel beam of light
falling on A leaves Bas a parallel beam, tnen distanced in
em will be

/
/

909

Focal Length
Power
(a) Decreased
increased
(b) Decreased
unchanged
(c) Increased
unchanged
(d) Increased
decreased
49. 1\vo convex lenses placed in contact form the image of a
distant object at P. If the lens B is moved to the right, the
image will

(a) 25
(c) 30

(b) 36
(d) 50

54. As shown in figure, the liquids L1, L2 and L 3 have refractive


indices 1.55, 1.50 and 1.20 respectively. Therefore , the

arrangement corresponds to

A B

(a)
(b)
(c)
(d)

move to the left


move to the right
remain ,a t P
move either to the left or right, depending upon focal
length of the lenses

(a) biconvex lens


(b) biconcave lens
(c) concavo-convex lens
(d) convexo-concave lens
55. A thin double convex lens has radii of curvature each of
magnitude 40 ern and is made of glass with fl = 1.65. The
focal length of the lens in nearly
(b) 31 ern
(a) 30 ern
(c) 40 ern
(d) 41 ern

910

Chapter 23 Ray Optics

56. A convex lens of focal length f produces a virtual image


n times the size of the object. Then the distance of the
object from the lens is

65. A convex lens is placed in contact with a mirror as shown


in figure. If the space between them is filled with water,
its power will

(a) (n- 1) f
(b) (n + 1) f
(c)

(n~1Jt

(d)

(n:1)f

57. A concave lens of focal length 20 em produces an image


half in size of the real object. The distance of the real
object is
(a) 20 em
(b) 30 em
(c) 10 em
(d) 60 em
58. An object 15 em high is placed 10 em from the optical
centre of a thin lens. Its image is formed 25 em from the
optical centre on the same side of the lens as the object.
The height of the image is
(a) 2.5 em
(b) 0 .2 em
(c) 16.7 em
(d) 37.5 em
59. One surface of a lens is convex and the other is concave.
If the radii of curvature are r1 and r 2 respectively, the lens
will be convex, if
(a) r 1 > r 2
(b) r 1 = r 2
(c) r 1

< r2

61. A convex lens of focal length I_ m forms a real, inverted


3
image twice in size of the object. The distance of the

63.

64.

67.

68.

(d) r 1 = 1/r2

60. A lens forms a virtual image 4 em away from it when an


object is placed 10 em away from it. The lens is a .... .lens
offocallength ...
(a) concave, 6.67 em
(b) concave, 2.86 em
(c) convex, 2.86 em
(d) May be concave or convex, 6.67 em

62.

66.

object form the lens is


(a) 0.5 m
(b) 0.166 m
(c) 0.33 m
(d) 1m
A object is placed at a distance offj2 from a convex lens
of focal length f. The image will be
(a) at one of the foci, virtual and double its size
(b) is greater than 1.5 but less than 2.0
(c) at 2f, virtual and erect
(d) None of the above
Consider an equiconvex lens of radius of curvature R
and focal length f. Iff> R, the refractive index I! of the
material of the lens
(a) is greater than zero but less than 1.5
(b) is greater than 1.5 but less than 2.0
(c) is greater than one but less than 1.5
(d) None of the above
A convex lens forms an image of an object placed 20 em
away from it at a distance of 20 em on the other side of
the lens . If the object is moved 5 em towards the-lens, the
image will move
(a) 5 em towards the lens
(b) 5 em away from the lens
(c) 10 em towards the lens
(d) 10 crrf away from the lens

69.

70.

71.

(a) decrease
(b) increase
(c) remain unchanged
(d) increase or decrease depending on the focal length
The power of a thin convex lens Cang = 1.5) is + 5.0 D.
When it is placed in a liquid of refractive index ani, then
it behaves as a concave lens of focal length 100 em. The
refractive index of the liquid ani will be
(a) 5/3
(b) 4/3
(c) F3
(d) 5/4
A concave lens with unequal radii of curvature made
of glass C~-t8 = 1.5 ) has a focal length of 40 em. If it is
immersed in a liquid of refractive index 1-ti = 2, then
(a) it behave like a convex lens of 80 em focal length
(b) it behave like a concave lens of 20 em focal length
(c) its focal length becomes 60 em
(d) nothing can be said
An object is put at a distance of 5 em from the first focus
of a convex lens of focal length 10 em. If a real image is
formed, its distance from the lens will be
(b) 20 em
(a) 15 em
(c) 25 em
(d) 30 em
A virtual image twice as long as the object is formed by a
convex lens when the object is 10 em away from it. A real
image twice as long as the object will be formed when it
is placed at a distance .... from the lens
(a) 40 em
(b) 30 em
(c) 20 em
(d) 15 em
An achromatic convergent doublet of two lenses in
contact has a power of+ 2 D . The convex lens has power
+5 D. What is the ratio of the dispersive powers of the
convergent and divergent lenses?

(a) 2 : 5
(b) 3 : 5
(c) 5 : 2
(d) 5 : 3
If i~Lj represents refractive index when a light ray goes from
medium ito medium j, then the product 2 ~-t 1 x 3 ~-t 2 x 4 ~-t3
is equal to
(a) 3~1
(c)

(b) 3 ~2

1
1~4

72. What. is the relation between refractive indices ~-t 1 , ~-t 2 , and
~-t 3 if the behaviour of light rays is as shown in figure.

(a) ~3 < ~2.~2 = ~1

(b) ~2

(c) ~3<~2 < ~1

(d) ~3 > ~2 > ~1

<

~1.~2 = ~3

Chapter 23 Ray Optics


73. Monochromatic light of wavelength A. 1 travelling in medium
of refractive index n 1 enters a denser medium of refractive
index n 2 The wavelength in the second medium is
(a) A1 (

::

(c) 1.1

(b) A1 (

~: J

(d) A1(

nz~n1 J

75. What is the angle of incidence for an equilateral prism of


refractive index /3 so that the ray is parallel to the base
inside the prism?
(b) 45
(a) 30
(d) Either 30 or 60
(c) 60
76. A lens of refractive index n is put in a liquid of refractive
index n'. If focal length of lens in air isf, its focal length in
liquid will be
(b) J(n' -n)
(a) fn'(n -1)
n'(n -1)
n'-n
n'(n-1)
fn'n
(c) ___:__ __:_

77.

(d)

n-n'

::~fJl:~:::~~~nili: ~ ; ; iiii:~::i;J
critical angle 8c. If thin layer of
water

(1-1 = ~ J is

l :

now poured on

'

the glass air surface, the angle at which the ray emerges
into air at the water-air surface is
(a) 60
(b) 45
(c) 90
(d) 180
78. Light is incident from a medium X at an angle of
incidence i and is refracted into a medium Y at angle of.
refraction r. The graph sin i versus sin r is shown in
figure. Which of the following conclusions would fit
the situation?
1. Speed of light in medium
Y is .J3 times that in
0.2
me diumX.
sin r
2. Speed of light in medium
sin i
Y is 1/ .J3 times that in
0.4
medium X.
3 . Total internal reflection will
occur above a certain i value.
(b) 1 and 3
(a) 2 and 3
(c) 2 only
(d) 3 only
79. When a glass slab is placed on a cross made on a sheet,
the cross appears raised by 1 em. The thickness of the
glass is 3 em. The critical angle for glass is
(a) sin- 1 (0.33)
(c) sin-1 ( .67)

(b) sin-:- 1 (0.5)


(d) sin- 1 (-h/2)

80. Monochromatic light of frequency 5 x 10 14 Hz travelling


in vacuum enters a medium of refractive index 1.5. It

wavelength in the medium is


(a) 4000 A
(b) 5000A
(c) 6000A
(d) 5500A

Optical Instruments

74. A bucket contains some transparent liquid and its depth is


40 em. On looking from above, the bottom appears to be
raised up by 8 em. The refractive index of the liquid is
(a) 5/4
(b) 5
(c) 4/5
(d) 8/5

j(n'-n)

911

81. The magnifying power of a telescope is 9. When it is


adjusted for parallel rays, the distance between the
objective and the eye-piece is found to be 20 em. The
focal lengths of the lenses are
(b) 11 em, 9 em
(a) 18 em, 2 em
(c) 10 em, 10 em
(d) 15 em, 5 em
82. In compound microscope, magnifying power is 95 and
1
the distance of object from objective lens is - - em.
1
3.8
The focal length of objective lens is - em. What is the
4
magnification of eye piece?
(a) 5
(b) 10
(c) 100
(d) 200
83. The focal lengths of the objective and eyelenses of a
microscope are 1.6 em and 2.5 em respectively. The
distance between the two lenses is 21.7 em. If the final
image is formed at infinity, the distance between the
object and the objective lens is
(a) 1.8 em
(b) 1.70 em
(c) 1.65 em
(d) 1.75 em
84. Two points, separated by a distance of 0.1 mm, can just
be inspected on a microscope when light of wavelength
6000 A is used. If the light of wavelength 4800 A is used,
the limit of resolution is
(a) 0.8 mm
(b) 0.08 mm
(c) 0.1 mm
(d) 0 .04 mm
85. The diameter of moon is 3.5 x 10 3 km and its distance
from the earth is 3.8 x 10 5 km. The focal length of the
objective and eye-piece are 4 m and 10 em respectively.
The diameter of the image of the moon will be
approximately
(a) 2
(b) 21
(d) 50
(c) 40
86. With diaphragm of the camera lens set atf/2, the correct
exposure time is 1/ 100 s. Then with diaphragm set atf/8,
the correct exposure time is
(b) l / 400 s
(a) l/100 s
(d) 16/100 s
(c) 1/200 s
87. An object is viewed through a compound microscope and
appears in focus when it is 5 mm away from the objective
lens. When a sheet of transparent material 3 mm thick
is placed between the objective and the microscope, the
objective lens has to be moved 1 mm to bring the object
back into the focus . The refractive index of the transparent
material is
(a) 1.5
(b) 1.6
(c) 1.8
(d) 2.0
88. A hypermetropic person having near point at a distance of
0.75 m puts on spectacles of power 2.5 D. The near point
now is at
(a) 0.75 m
(b) 0.83 m
(c) 0.26 em
(d) 0.26 m

912

Chapter 23 Ray Optics

89. An astronomical telescope has a converging eye-piece of


focal length 5 ern and objective of focal length 80 em. When

the final image is formed at the least distance of distinct


vision (25 em), the separation between the two lenses is
(a) 75.0 em
(b) 80.0 em
(d) 85.0 em
(c) 84.2 em
90. The focallengthofobj ective and eye lensof:mastronomical
telescope are respectively 2 m and 5 em. Final image is
formed at (1) least distance of distinct vision (2) infinity.
Magnifying powers in two cases will be
(a) - 48,-40
(b) - 40, 48
(c) -40, + 48
(d) -48, + 40
91. A man's near point is 0.5 m and far point is 3 m. Power
spectacle lanses repaired for
(i) reading purposes
(ii) seeing distant objects, respectively.
(a) -2 D and + 3D
(b) + 2 D and- 3 D
(c) + 2 D and- 0.33 D
(d) -2 D and + 0.33 D
92. A hypermetropic person has to use a lens of power + 5 D to
normalise his vision. The near point of the hypermetropic
eye is
(b) 1.5 m
(a) 1m
(d) 0.66 m
(c) 0 .5 m
93. A compound microscope has an objective and eye-piece
as thin lenses of focal lengths 1 em and 5 em respectively.
The distance between the objective and the eye-piece is
20 em. The distance at which the object must be placed
infront of the objective if the final image is located
at 25 em from the eye-piece, is numerically
(u) 95/6 em
'
(b ) 5 em
(c) 95/ 89 em
(d) 25/6 em
94. The focal length of the objective and the eye-piece of a
microscope are 4 mm and 25 mm respectively. If the final
image is formed at infinity and the length of the tube is
16 em, then the magnifying power of microscope will be
(a) -337.5
(b) -3.75
(c) 3.375
(d) 33. 75
95. A simple microscope consists of a concave lens of power
- 1 OD and a convex lens of power + 20D in contact. If the
image is formed at infinity , then the magnifying power
CD= 25 em is
(a) 2.5
(b ) 3 .5
(d) 3.0
(c) 2.0
96. The magnifying power of an astronomical telescope is 10
and the focal length of its eye-piece is 20 em. The focal
length of its objective will be
(b) 2 em
(a) 200 em
(c) 0 .5 em
(d) 0 .5 x 10-2 em

(a) 30

(c) 60
99. It is desired to make a converging achromatic combination
of mean focal length 50 em by using two lenses of
materials A and B. If the dispersive powers of A and B
are in ratio 1: 2, the focal lengths of the convex ar.d the

100.

101.

102.

103.

concave lenses are respectively


(b) 50 em and 25 em
(a) 25 em and 50 em
(c) 50 em and 100 em
(d) 100 em and 50 em
Two parallel light rays are incident at one surface of a
prism of refractive index 1.5 as shown in figure. The
angle between the emergent rays is nearly
(a) 19
(b) 37
(c) 45
(d) 49
The refractive index of the material of a prism is J2 and
the angle of prism is 30. One of its refracting faces is
polished. The incident beam of light will retrace back for
angle of incidence
(a) 0
(b) 45
(c) 60
(d) 90
The cross-section of a glass prism has the form of an
isoceles triangle. One of the refracting faces is silvered.
A ray of light falls normally on the other refracting face .
After being reflected twice, it emerges through the base of
the prism perpendicular to it. The angles of the prism are
(a) 54, 54, no
(b) no, no, 36
(c) 45, 45, 90
(d) 57, 57, 76
Parallel beam containing light of'), = 400 nm and 500 nm
is incident on a prism as shown in figure. The refractive
index fl of the prism is given by the relation

1-!C"-) = 1.20 + o.s x ~o-14


I.
10
Which of the following statement
is correct?
,'
(a) Light on= 400 nm undergoes
total internal reflection.
(b) Light of A. = 500 nm undergoes
total internal reflection.
(c) Neither of the two wavelengths undergoes total
internal reflection.
(d) Both wavelengths undergoes total internal reflection.
104. The maximum refractive index of a prism which permits
the passage of light through it, when the refracting angle
of the prism is 90, is
(a)

J3

(b)

J2

(c)

J3

(d)

Prism
97. For a prism, its refractive index is cos A. Then minimum
2
angle of deviation is
(a) 180 -A
(b) 180- 2A
(c) 90 - A

(d) A
2
98. The refractive index of a prism for a monochromatic
wave is J2 and its refracting angle is 60. For minimum
deviation, the angle of incidence will be

105. An object is placed 30 em to the left of a diverging lens


whose focal length is of magnitude 20 em. Which one of
the following correctly states the nature and position of
the virtual image formed?
Nature of image
Distance from lens
(a) inverted, enlarged
60 em to the right
(b) erect, diminished
12 em to the left
(c) inverted, enlarged
60 em to the left
(d) erect, diminished
12 em to the right
(e) inverted, enlarged
12 em to the left

Chapter 23 Ray Optics

913

Exercise II
Only One Correct Option
1. A ray oflight passes through four transparent medium with
refractive indices J.11, J.12, J.13 and J.14 as shown in the figure.

The surfaces of all media are parallel. If the emergent ray


CD is parallel to the incident ray AB. we must have
1!3

(a) tan- 1 (1.62)


(c) cos- 1(1.62)

(b) sin-1 (1.62)


(d) None of these

9. A double convex lens made out of glass (refractive index


J.1 == 1.5) has both radii of curvature of magnitudes 20 em.

Incident light rays parallel to the axis of this lens will


converge at a distance d such that
20
(a) d = 10 em
(b) d =-em
3

(a) J.11 == J.12


(c) J.13 == J.l4

(b) J.12 == J.l3


(d) J.13 == J.11

2. A diminished image of an object is to be obtained on


a screen 1.0 m away from it. This can be achieved by
approximately placing
(a) a convex mirror of suitable focal length
(b) a concave mirror of suitable focal length
(c) a convex lens of focal length less than 0.25 m
(d) a concave lens of suitable focal length
3. A ray of light passes through an equilateral prism such
that the angle of incidence and the angle of emergence
are both equal to 3/ 4 th of the angle of prism. The angle
of minimum deviation is
(a) 15
(b) 30
(c) 45
(d) 60
4. A lens of focallengthf projects i'rt:tiines magnified image
of an object on .a scnen. the distance of the screen from
the lens is

(m~ 1)

(a)
(c)

(m -1)

(c) d == 40 em
(d) d == 20 em
10. A 4 em thick layer of
,--,...,---,----water covers a 6 em 4cm
thick glass slab. A coin
is placed at the bottom
of the slab and is being
observed from the air
side along the normal
Coin
to the surface. Find _the
apparent position of the coin from
(a) 7.0 em
(b) 8.0 em
(c) 10 em
(d) 5 em
11. The light takes in travelling a distance of 500 m in
water. Given that J.1 for water is 4/ 3 and the velocity of
light in vacuum is 3 x 10 10 cms-1 Calculate equivalent
optical path.
(a) 566.64 m
(b) 666.64 m
(c) 586.45 m
(d) 576.64 m
p
12. How many images are formed by the lens shown, if an
object is kept on its axis?

(b) (m: 1)
(d)

(m

+ 1)

1!1

5. A thin equiconvex lens of refractive index 3/ 2 and radius


of curvature 30m is put in water (refractive index = ~).
Its focal length is
(a) 0.15 m
(b) 0.30 m
(c) 0.45 m
(d) 1.20 m
6. A 16 em long image of an object is formed by a convex
lens on a screen. On moving the lens towards the screen,
without changing the positions of the object and the
screen, a 9 em long image is formed again on the screen.
The size of the object is
(a) 9 em
(b) 11 em
(c) 12 em
(d) 13 em
7. TWo lenses, one concave and the other convex of same
power' a:re placed such that their principal axes coincide.
If the separation between the lenses is x, then
(a) real image is formed for X== o<&tily
(b) real image is formed for all vafues of x
(c) system will behave like a glass-~late- for x = 0
(d) virtual image is formed for a'liSV.alues of x other than
.
.Jr.
zero
'
8. A ray of light falls on a transparent glass sl~b with
refractive index (relative to air) of 1.62. The angle of
incidence for which the reflected and refracted rays are
mutually perpendicular is

(a) 1
(c) 3

(b) 2

(d) 4

13. For a optical arrangement shown in the figure. Find the


position and nature of image.

(a) 32 em
(c) 6 em

(b) 0.6 em
(d) 0.5 em

14. A thin plano-convex lens of focal length f is split into two


halves. One of the halves is shifted along the optical axis.
The separation between object and image plane is 1.8 m.
The magnification of the image fonned by one Of the half
lens is 2. Find the focal-length of the lens and separation
between the two halves.

914

Chapter 23 Ray Optics

(b) 0.4 m
(d) 1m

(a) 0.1 m
(c) 0.9 m

15. The refractive indices of the crown glass for blue and red
light are 1.51 and 1.49 respectively and those of the flint
glass area 1. 77 and 1. 73 respectively. An isosceles prism
of angle 6 is made of crown glass. A beam of white light
is incident at a small angle on this prism. The other flint
glass isosceles prism is combined with the crown glass
prism such that there is no deviation of the incident
light. (i) Determine the angle of the flint glass prism.
(ii) Calculate the net dispersion of the combined system.
(a) -4, 0.04,
(b) 4, 0 .04
(c) 5, 0.04
(d) -5, 0 .04
16. A plano-convex lens has a thickness of 4 em. When placed
on a horizontal table, with the curved surface in contact
with it, the apparent depth of the bottom most point of the
lens is found to be 3 em. If the lens is inverted such that the
plane face is in contact with the table, the apparent depth of
uJ L the centre of the plane face is found to be 25/ 8 em. Find the
focal length of the lens. Assume thickness to be negligible
(a) 85 em
(b) 59 em
(d) 7.5 em
(c) 75 em
17. A convex lens of focallengthf is placed some where in
between an object and a streen. The distance between
object and screen is x. If numerical value of magnification
produced by lens is m, focal length of lens is
to w,
mx
mx
(b)
2r br(a)
2
2

(c)

(d)

(m-1f

19. If eye is kept at a depth h inside water of refractive


.
d outs1"de, then t h e d"1ameter of t h e crrc
. le
and VIewe
1
,r ' through which the outer objects become visible, will be

~~2 -1

ri!gn9J'. .

suld 'c) ~

(b) _ h _

~~2+1

2h
2

(d)

{1 -~ J

23. A point object is placed at a distance of '25 em from


a convex lens of focal length 20 em. If a glass slab of
thickness t and refractive index 1.5 is inserted between
the lens and the object, the image is formed at infinity.
The thickness t is
(a) 15 em
(b) 5 em
(c) 10 em
(d) 20 em
24. A plano convex lens fits exactly into a plano concave lens.
Their plane surfaces are parallel to each other. If the lenses
are made of different materials of refractive indices f1 1 and
f.l 2 and R is the radius of curvature of the curved surface of
the lenses, then focal length of the combination is
R

(a)

(b)

2(~1

21 2
, [{ m: d ex

t (1-~ J

(d) awayfromPby

(c)

18. A ray oflight from a denser medium strikes a rarer medium


at angle of incidence i. The reflected and refracted rays
make an angle of 90 with each other. The angles of
j f::.J:.!eflection and refraction are r and r' respectively. The
2
critical angle is
(a) sin-1(tanr')
(b) sin- 1(tanr)
(c) tan-1(tanr')

(a) _ h _

(c) towardsPby

+ ~2)
R

(m-1)

(m+1f
(m+ 1)

21. An object is kept at a distance of 16 em from a thin lens


and the image formed is real. If the object is kept at a
distance of 6 em from the same lens, the image formed
is virtual. If the sizes of the images formed are equal the
focal length of the lens will be
(b) 11 em
(a) 21 em
(c) 15 em
(d) 17 em
22. P is a point on the axis of a concave mirror. The image of
P formed by the mirror, coincides with P. A rectangular
glass slab of thickness t and refractive index f.l is now
introduced between P and the mirror. For the image of P
to coincide with P again, the mirror must be moved
(a) towardsPby (~-1)t
(b) awayfromPby (~-1)t

(~1 - ~2 )

2(~1- ~2)

(d)

2R
(~2 -~1)

25. One of the refracting surfaces of a prism of angle 30 is


silvered. A ray of light incident at an angle of 60 retraces
its path. The refractive index of the material of prism is
(a)

(c) 2

(b) 3/2
(d)

ifi

26. The focal length of objective and eye-piece of a microscope


are 1 em and 5 em respectively. If the magnifying power
for relaxed eye is45, then length of the tube is
(a) 9 em
(b) 15 em
(d) 6 em
(c) 12 em
27. A glass prism ABC (refractive index 1.5) , immersed in
water (refractive index 4/3) . A ray of light is incident
normally on face AB. If it is totally reflected at face AC
then

~ -1
vw
20. A ray of light is incident at 60 on one face of a prism
whi.ch has angle 30. The angle between the emergent ray
and incident ray is 30. What is the angle between the ray
and the face from. which its emerges?
(a) 0
(b) 30
(c) 60
(d) 90

-:_-------. c _-_-_-_-_-_-_-_-_-_-.::.::_-_-_-.

915

Chapter 23 Ray Optics

. e 2':-2

(a)

Sill

. e 2':-8

(b)

Sill

(c)

sine= F3

(d)

- <Sill 9 < -

2
3

9
28. A plane mirror is placed at the bottom of a tank containing
a liquid of refractive index fl P is a small object at a
height h above the mirror. An observer 0 -vertically above
P outside the liquid sees P and its image in a mirror. The
apparent distance between these two will be
2

o.
I

I
I
I
I

(a) When a ray of light travels from 3 to 1 no TIR will


take place
\
(b) Critical angle between 1 and 2 is less than the critical
angle between 1 and 3
(c) Critical angle between 1 and 2 is more than the critical
angle between 1 and 3
(d) Chances ofTIR are more when ray oflight travels from
1 to 3 compare to the case when it travel from 1 to 2
34. Parallel rays oflight are falling on convex spherical surface
of radius of curvature R = 20 em as shown. Refractive
index of the medium is fl = 1.5 . After refraction from the
spherical surface parallel rays

~
(a) 2 fl h
(c)

_3!!_
J.L-1

29. Light takes t 1 second to travel a distance x in vacuum and


the same light takes t 2 second totravel10xcminamedium.
Critical angle for corresponding medium will be

(a) sin-1 (

1~1tz J

(a) actually meet at some point


(b) appears to meet after extending the refracted rays
backwards
(c) meet (or appears to meet) at a distance of 30 em from
the spherical surface
(d) meet (or appears to meet) at a distance of 60 em from
the spherical surface
35. A ray of light travelling in a transparent medium falls on
a surface separating the medium from air, at an angle of
incidence of 45. The ray undergoes total internal reflection.
If n is the refractive index of the medium with respect to
air, select the possible values of n from the following
(a) 1.3
(b) 1.4
(c) 1.5
(d) 1.6
.'\f

1
. -1(10t
(c) sm
-t2

(d) sm
. -1( -t110t2

Comprehension Based Questions


Passage I
I

30. A plano convex lens of (f = 20 em) is silvered at plane

surface. New fwill be


(a) 20 em
(c) 30 em

(b) 40 em
(d) 10 em

May have More than One Correct Option


31~ A point object is at 30 em from a convex glass lens

(lls =%) of focal length 20 em. The final image of object


will be formed at infinity if
(a) another concave lens of focal length 60 em is placed
in contact with the previous leJ:lS
(b) another convex lens of focal length 60 em is placed at
a distance of 30 em from the first lens
(c) the whole system is immersed in a liquid of refractive
,.
index 4/3
(d) the .w hole system is immersed i:p. fi liquid of refractive
index 9;8
... u.

32. For which of the pairs of u and f for a mirror image is


smaller in size.
(a) u = - 10 em, f = 20 em
(b) u = - 20 em, f = - 30 em
(c) u =- 45 em, f = -10 em
(d) u =- 60 em, f = 30 em
33. There are three optical media, 1, 2 and 3 with their refractive
indices fll j> f.!z > f.! 3. (TIR- total internal reflection)

The power of a convex lens depends on the radius of


curvature and refractive index of lens material a'nd is
given by
Pa = Ca llg -1) (

~1 - ~2

when lens is in air.

The refractive index of material is roughly.given by JH


B
~
Jl =A+ /.} (Cauchy's formula).
1
If a lens is dipped in a liquid, its power is changed and is
given by

~ = C;J.tg -1)( ~1 ~J
-

J:;

36. A lens (f.!g=l.5) in air has a power + 2 D. When l~n~fs


dipped in water of refractive index 4/3, its powe~ will
1
become/remain
(a) + 4 D
(c)+ 2D

(b) + 8 D
(d) -4D

"'

3 7. The refractive index of glass for yellow light of wavelength


6000 Ais 1.5. Then the refractive index of glass fo1 blue
light A = 4000 A will become/remain
(a) 1.5
(b) less than 1.5
(d) information is insufficient
(c) greater than 1.5
38. If a hollow convex shaped glass is filled with water and
surrounding is glass. The lens will act as
(a) convex lens
(b) concave lens
(c) glass plate
(d) convex mirror

916

Chapter 23 Ray Optics

Passage II
Total internal refl'ection is the phenomenon of reflection
of light into denser medium at the interface of denser
medium with a rarer medium. Light must travel from
denser to rarer and angle of incidence in denser medium
must be greater than critical angle (C) for the pair of
media in contact. We can show that
1

11 =sine
39. Critical angle for water air interface is 48.6. What is the
refractive index of water?
(a) 1
(b) 3/2
(c) 4/3
(d) 3/ 4

40. Light is travelling from air to water at Li = 50, which


is greater than critical angle for air water interface. What
fraction of light will be totally reflected?
(b) 50%
(a) 100%
(c) None
(d) Cannot say

41. Critical angle for glass air interface where J.! of glass is 3/ 2 is
(a) 41.8
(b) 60
(c) 30
(d) 44.3
42. Critical angle for air water interface for violet colour is
49. Its value for red colour would be
(a) 49
(b) 50
(c) 48
(d) cannot say
43. A point source of light is held at a depth h below the
surface of water. If Cis critical angle of air water interface,
the diameter of circle of light coming from water surface
would be
(b) h tan C
(a) 2 h tan C
(d) h/ sin C
(c) h sin C

Assertion and Reason


Directions Question No . 44 to 50 are Assertion-Reason
type. Each of these contains two Statements : Statement I
(Assertion), Statement II (Reason). Each of these questions
also has. four alternative choice, only one of which is correct.
You have to select the correct choices from the codes (a) , (b),
(c) and (d) given below:
(a) If both Assertion and Reason are true and the Reason is
correct explanation of the Assertion.
(b) If both Assertion and Reason are true but Reason is not
correct explanation of the Assertion.
(c) If Assertion is true but Reason is false .
(d) If Assertion is false but the Reason is true.
0
44. Assertion Convergent lens property of converging remain
same in all mediums.
Reason Property of lens whether the ray is diverging or
converging depends on the surrounding medium.
45. Assertion A concave mirror and convex lens both have
' the same focal length in air. When they are submerged in
water, they will still have the same focal length.
Reason The refractive index of water is greater than the
refractive index of air.
46. Assertion The focal length of the objective of the telescope
is larger than that of eye-piece
Reason The resolving power of telescope increases when
the aperture of objective is small.
~

47. Assertion A short sighted person cannot see objects


clearly when placed beyond 50 em. He should use a
concave lens of power 2 D.
Reason Concave lens should form image of an object at
infinity placed at a distance of 50 em.

48. Assertion By roughening the surface of a glass sheet its


transparency can be reduced.
Reason Glass sheet with rough surface absorbs more
light.
49. Assertion The refractive index of diamond is /6 and
that of liquid is .J3 . If the light travels from diamond
to the liquid, it will initially reflected when the angle of
incidence is 30.
1
Reason J.! = - .- - where J.! is the refrective index of
smC
diamond with respect to liquid.
50. Assertion The colour of the green flower seen through
red glass appears to be dark.
Reason Red glass transmits only red light.

Previous Year's questions


51. The distance between an object and a divergent lens ism
times the focal length of the lens. The linear magnification
produced by the lens is
(WB JEE 2009)
(a) m
(b) 1/m
(c) (m

(d)

1)

m+1
52. In an optics experiment, with the position of the object
fixed, a student varies the position of a convex lens and
for each position, the screen is adjusted to get a clear
image of the object. A graph between the object distance
u and the image distance v, from the lens, is plotted using
the same scale for the two axes. A straight line passing
through origin and making an angle of 45 with the x-axis
meets the experimental curve at P. The coo~dinate of P
will be
(AIEEE 2009)
(a) (

f.f)

(b) (j,j)

(c) (4j, 4f)

53. A thin lens of ( J.!

immersed in water (J.!

(d) (2/, 2/)

1.5) of focal length + 10 em is


1.33). The new focal length is

(DCE 2009)

(a) 20 em
(c) 48 em

(b) 40 em
(d) 12 em

54. A telescope consists of two thin lenses of focal lengths


0.3 m and 3 em respectively. It is focused on moon which
subtends an angle of 0.5 at the objective. Then, the angle
subtended at the eye by the final image will be
(UP SEE 2009)

(a) 5

(c) 0.5

(b) .0.25
(d) 0.35

55. A ray oflight passes through an equilateral prism such that


the angle of incidence is equal to the angle of emergence
and the latter is equal to ~ the angle of prism. The angle
4
of deviation is
(UP SEE 2009)
(a) 25
(bJ--:50
(c) 45
(d) 35

Chapter 23 Ray Optics


56. Two plane mirrors are inclined at an angle e. It is found
that a ray incident on one mirror at any angle is rendered
parallel to itself after reflection from both the mirrors.
The value of 8 is
(UP SEE 2009)
(a) 30
(b) 60
(c) 90
(d) 120
57. When a ray of light enters a glass slab from air
(UP SEE 2009}

(a) its wavelength decreases


(b) its wavelength increases
(c) its frequency increases
(d) Neither its wavelength nor its frequency changes
58. Critical angle of light passing from glass to water is
minimum for
(UP SEE 2009)
(a) red colour
(b) green colour
(c) yellow colour
(d) violet colour
59. In a spectrometer experiment, "as wavelength of a spectral
line increases, deviation also increases." This is true

66. If the angle of minimum deviation is 60 for an equilateral


prism, then the refractive index of the material of the
prism is
(Kerala CET 2008)
(a) 1.41
(b) 1.5
(c) 1.6
(d) 1.33
67. A student measures the focal length of a convex lens by
putting an object pin at a distance u from the lens and
measuring the distance v of the image pin. The graph
between u and v plotted by the student should look like
(AIEEE 2008)

(a)

(Karnataka CET 2006}

(b) 0.09 m, 0.18 m


(a) 0.045m, 0.09 m
(d) 0.06m, 0.12 m
(c) 0.04m, 0.08 m
61. A person inside water llw = ' 4/3 sees the setting sun at
about
(Karnataka CET 2008)
(a) 42 to the horizontal
(b) 48 to the horizontal
(c) 42 to the vertical
(d) 24 to the vertical
62. A ray of light is incident on a glass slab of thickness t, at
an angle i, r is the angle of refraction in the glass plate.
Distance travelled in the glass plate is
(Karnataka CET 2008)

(a) t cos r
(b) ttanr
(c) t/cos r
(d) t/sin r
63. A right angled hollow prism is filled with a liquid. A ray
of light entering the prism grazing one face emerges out
grazing other face. Refractive index of the liquid is
(Karnataka CET 2008)

(b) 1.33
(c) 1.54
(d) 1.41
64. The radius of curvature of the convex face of a
plano-convex lens is 15 em and the refractive index of the
material is 1.4. Then the power of the lens in diopter is
(a) 1. 73

(Kerala CET 2008)

(a) 1.6
(b) 1.66
(d) 2.66
(c) 2.6
65. The wavelength of red light from He-Ne laser is 633 nm in
air but 474 nm in the aqueous humor inside the eye ball.
The speed of red light through the aqueous humor is
(Kerala CET 2008)

(a) 3 x 108 ms-1


(c) 2.25 x 108 ms-1

(b) .1.34 x 108 ms-1


(d) 2.5 x 10 8 ms-1

v(cm)

(c)

v(cm)

t
(b)

(Karnataka CET 2008)

(a) in grating spectrum


(b) in prism spectrum
(c) Both in grating as well as prism spectrum
(d) Neither in prism spectrum nor in grating spectrum
60. The focal length of a biconvex lens of refractive index 1.5
is 0.06 m. Radius of curvature are in the ratio 1 : 2. Then
radii of curvatures of two lens surface are

917

u(cm)

- "

) tm)

u(cm)

'IT ~

t
0

u(cm)

u(cm)

(d)

68. An experiment is performed to find the refractive index


of glass using a travelling microscope. In this experiment
distances are measured by
(AIEEE 2008)
(a) a screw gauge provided on the microscope
(b) a vernier scale provided on the microscope
(c) a standard laboratory scale
(d) a metre scale provided on the microscope
69. Given allg = 3/2 and allw = 4/3. There is an equiconvex
lens with radius of each surface equal to 20 em. There is
air in the object space and water in the image space. The
focal length of lens is
(DCE 2008)
(a) 80 em
(b) 40 em
(c) 20cm
(d) 10 em
70. An observer looks at a tree of height 15m with a telescope
of magnifying power 10. To him the tree appears
(BVP Engg. 2008)

(a) 10 times taller


(b) 15 times taller
(c) 10 times nearer
(d) 15 times nearer
71. The resolution limit of the eye is 1 min. At a distance x km
from the eye, two persons stand with a lateral separation
of 3m. For the two persons to be just resolved by the
naked eye, x should be
(BVP Engg. 2008)
(a) 10 km
(b) 15 km
(c) 20 km
(d) 30 km
72. A thin prism P 1 with angle 4 and made from glass of
refractive index 1.54 is combined with another thin prism
P2 made from glass of refractive index 1.72 tb produce
dispersion without deviation. The angle of the prism P2 is
(BVP Engg. 2008)

(a) 5.33
(c) 3
73. The image formed by a concave mirror
(UP SEE 2008)
(a) is always real
(b) is always virtual
(c) is certainly real if the object is virtual
(d) is certainly virtual

II

.9t8

Chapter 23 Ray Optics

------

~~~A lens

made of glass whose index of refraction is 1.60 has


a focal length of + 20 em in air. Its focal length in water,
', whose refractive index is 1.33, will be
(UP SEE 2008)
(a) three times longer than in air
(b) two times longer than in air
(c) same as in air
(d) None of the above
(~5 . Magnification at least distance of distinct vision of a
(B) simple microscope having its focal length 5 em is

(Kerala CET 2007)

(::>)

(L)
(d)
(B)

..

(a) 2
(c) 5
(e) 7

(b) 4
(d) 6

(' ,

76. The limit of resolution of an optical instrument arises on


lL account of
(Gujarat CET 2007)

(B)

(d)

(a) reflection
(c) polarization

(b) diffraction
(d) interference

77. The length of deviation for a glass prism is equal to its


refracting angle. The refractive index of glass is 1.5 . Then
the angle of prism is
(J & K CET 2007)
(E)
(a) 2 cos- 1 (3/ 4)
(b) sin- 1 (3/ 4)
li') (c) 2 sin- 1 (3/ 2)
(d) cos-1 (3/2)

~.

9
(.

A convex lens is placed between object and a screen.

::> J
1

'The size of object is 3 em and an image of height 9 em is


obtained on the screen. When the lens is displaced to a new
lB) position, what will be the size of image on the screen?
'3

(Orissa JEE 2007)

(a) 2 em
(c) 4 em

(b) 6 em
(d) 1 em

79. A glass slab (J.! = 1.5) of thickness 6 em is placed over a


paper. What is the shift in the letters?
(DCE 2007)
(a) 4 em
(b) 2 em
cc) 1 em
(d) None of these
80. In a laboratory four convex lenses L1 , L2 , L 3 and L4 of
focal lengths 2, 4, 6 and 8 em, respectively are available.
Two of these lenses form a telescope of length 10 em and
magnifying power 4. The objective and eye lenses are
respectively
(UP SEE 2007)
(a) L2 , L3
(b) L1 , L4
(c) L1, L2
(d) L4 , L1

81. A symmetric double convex lens is cut in two equal parts

by a plane perpendicular to the principle axis. If the


power of the original lens is 4 D, the power of a cut lens
will be
(UP SEE 2007)
(a) 2 D
(c) 4D

(b)- 3D
(d) 5D

82. Focal length of objective and eye piece of telescope are


200 em and 4 em respectively. What is the length of
telescope for normal adjustment?
(DCE 2006)
(a) 196 em
(b) 204 em
(c) 250 em
(d) 225 em
83. The refractive index of glass is 1.520 for .Jed light and
1.525 for blue light. Let D1 and D2 be the angles of

minimum deviation for red light and blue light respectively


{A IEEE 2006)
in a prism of this glass. Then
(a) D1 > D2
(b) D 1 = D 2
(c) D 1 = D 2
(d) D 1 can be less than or greater than 9-epending upon

the angle of the prism


84. A point object is placed at the centre of a glass sphere of

radius 6 em and refractive index 1.5. The distance of the


virtual image from the surface of sphere is
(BHU 2006)
(a) 2 em
(b) 4 em
(c) 6 em
(d) 12 em
85. The minimum diameter of the cross-section of cone of
light with surface of water due to luminous point 8 ern
below the surface is (J.! = 1.33)
(BVP Engg. 2006)
(a) 18.24 em
(b) 20.24 em
(c) 25 em
(d) 30 em
86. Light of certain colour has 200 waves to the millimeter in
air. What will be the wavelength of this light in medium
of refractive index 1.25?
(BVP Eng g. 2006)
(a)

1ooo A

Cb) 2ooo A

Cc) 3ooo A
Cd) 4ooo A
87. A light moves from denser to rarer medium. Which of the
following is correct?
(UP SEE 2006)
(a) Energy increases
(b) Frequency increases
(c) Phase changes by 90 (d) Velocity increases

Chapter 23 Ray Optics

I .~919

Answers
Exercise I
1.
11.
2;1.
31.
41.
51.
61.
71.
81.
91.
101.

(a)
(c)
(a)
(a)
(a)
(d)
(a)
(c)
(a)
(c)

(b)

2.
12.
22.
32.
42.
52.
62.

72.
82.
92.
102.

(c)
(c)
(c)
(c)
(a)
(a)
(a)
(a)
(a)
(a)
(b)

3.
13.
23.
33.
43.
53.
63.

73.
83.
93.
103.

(b)
(c)
(c)
(d)
(a)
(b)
(c)
(a)
(d)
(c)
(a)

4.
14.
24.
34.
44.
54.
64.
74.
84.
94.
104.

5.
15.
25.
35.
45.
55.
65.
75.
85.
95.
105.

(c)
(a)
(a)
(d)
(b)
(c)
(d)
(a)
(b)
(a)
(b)

(b)
(c)
(d)
(b)
(b)
(b)
(a)
(c)
(b)
(a)
(b)

6.
16.
26.
36.
46.
56.
66.
76.
86.
96.

7.
17.
27.
37.
47.
57.
67.
77.
87.
97.

(d)
(a)
(a)
(c)
(b)
(c)
(a)
(a)
(d)
(a)

(a)
(b)
(c)
(c)
(d)
(a)
(a)
(c)
(a)
(b)

8.
18.
28.
38.
48.
58.
68.
78.
88.
98.

(c)
(c)
(a)
(b)
(d)
(d)
(d)
(c)
(d)
(b)

9.
19.
29.
39.
49.
59.
69.
79.
89.
99.

(b)
(b)
(b)
(a)
(b)
(c)
(b)
(c)
(c)
(a)

10.
20.
30.
40.
50.
60.
70.
80.
90.
100.

~)

(a)
(c)
(d)
(b)
(a)

\h)
(a)
(a)
(b)
\

Exercise II
1.
11.
21.
31.
41.
51.
61.

(a)

(b)
(b)

(a,d)
(a)
(d)
(a)
71. (a)
81. (a)

(c)
(a)
(c)
(a,c,d)
(c)
(d)
(c)
72. (c)
82. (b)

2.
12.
22.
32.
42.
52.
62.

3. (b)
(b)
(a)
(a,c,d)
(a)
(b)
(d)
73. (c)
83. (a)

13.
23.
33.
43.
53.
63.

4.
14.
24.
34.
44.
54.
64.
74.
84.

(d)
(b)
(c) '
(a,d)
(d)
(a)
(d)
(a)
(c)

5.
15.
25.
35.
45.
55.
65.
75.
85.

(d)
(a)
(b)
(c,d)
(d)
(b)
'(c)
(d)
(a)

6.
16.
26.
36.
46.
56.
66.
76.
86.

(c)
(c)
(b)
(b)
(d)
(c)
(e)
(b)
(d)

7.
17.
27.
37.
47.
57.
67.
77.
87.

(b,~)

(a)
(a)
(c)
(a)
(a)
(d)
(a)
(d)

8.
18.
28.
38.
48.
58.
68.
78.

(a)
(b)
(b)
(b)
(c)
(d)
(b)
(d)

9.
19.
29.
39.
49.
59.
69.
79.

(d)
(c)
(c)
(c)
(a)
(d)
(b)

(b)

. 10. (a)
20. (d)
30. ~~
40. c)
50. (a)
60. (a)
70. (c)
80. (d)

_~hapter 23

920

Ray Optics

Hints & Solutions


l
w 9't.
til{

Exercise I

p-

~.AU

E=~=-I-=~
o
r2 (4)2 16

3.

E = Icos8 = I x (415)
p
r' 2
C5i

r3

12. The variation of relative luminosity with wavelength is


shown here .
1.0

Relative
luminosity

Icos60
200x200
or
I= 5 x 1o- 4 x 4 x 10 4 x 2 = 40 cd
r =40- x-40
16
6. -=I
50 x 50 25

5.

E cx: -1

or

41
125
E
I 125 125
:. -0= - X- = Ep 16 4I
64
4. At minimwn distance, incidence is normal. Therefore,
I
250
E=-z==6.94 lux.
2

5 x 1o- 4

400

700

Wavelength (in nm) -

1-E.= 1 - 16 =_2_
I
25 25
I-I'
9
or
- - x 100=- x 100=36o/o
I
25
7. When the screen is equally illuminated,
E1 = E2

!J_ = .!1_ or !J_ = r/ = 30 x 30 = _2_

or

r1

r]

I2

ri

50 x SO

25

1
16
8. 502 =d2

d2 = (50i x 16

or

or
d=50 x 4cm=200cm=2m.
9. r 1 = 10 em, r 2 = 8 em
I1

I2

64
I
1 1
100 ' - I
2

or
or
1
10. x z

_
-

I 2 -I1 = 36
I 2 o 100

Iz-IJ x 100 '=36o/o


I2

16
(IOO-x)2
1

or

x
or

64
1 - 100

4
100-x

Sx =-100 or X= 20 em

Note that

360
.8

is odd and object line asymmetrically.

1 1 1
1
1 1
17. -+-=-or-=--+-

Now, compare withy = mx + c


Therefore graph is a straight line having negative slope.
18. Think in terms of rectangular hyperbola.
19. The angular range is clearly twice the critical angle .
1
1
1
1 31
20. - + - - = - o r - = v -600 20
v 600
600
or
v =-em= 19.35
31

Chapter 23 Ray Optics

or

-2=~

21. _!_=_!_
f-u 4

f -(-0.5)

4f=f+0.5 or 3/=0.5

0.5

or

f =-m=0.17m
3

20+u
or
20+u = -10
or
u =-30cm.
29. Clearly, the given mirror is a convex mirror.
m =_f___

f-u

22. The image is erect and diminished. So, the mirror in


necessarily convex.

1
1 1
23. - + - = -30 v 30
1
2
1
or
v 30 15
or
v = 15 em
06
24. f =- = -0.3m = -30cm
2
1
1
1
-+- =v -10 -30

1
18
3 18-u
3x18 = 18-u
u=-2 x 18 cmoru =-36crn.
-=--

or
or

1
1t
30. 8=-x-rad
2 180

1
v

1
1
3-1
10 30
30
30
or
v = - em= 15 em
2
v
15
m = -- = - - = 1.5
u
-10
Object lies between principal focus and pole. So, the
image is virtual and erect.
1 1 1
25. -+-=u v f
1
1
1
--+--=f-xl f-x2 f
f-x 2 + f-x 1 _ 1
Cf-xl)(f-x2) -~

or

diameter of image =
8
focal length
or diameter of image
1
1t
15
=-x-x-x100cm=6 55cm
2 180 2
.
.
31. For concave mirror
15
u=--cm
2
' v="'.
10
f =--em= -Scm
2
1 1 1 1
1
=---=----v f
u -5 -15/2.
1 2
-1
=--+-=5 15 15
or
v = -15 em
Clearly; the position of the final image is on the pole of
the convex mirror.
32. f = -10 ern, 0 = 5 em,
u = - 100 em, I = ?

!__ =_f___

2
f - fx2- fx1 + X1X2 = 2/ 2 - f(xl + X2)
2
or
f = x 1x 2 or f = Jx1x 2
This is Newton's mirror formula .
or

f-u

-10
-10
xs = -xs ern
-10-(-100)
90
= -0.55 em

I=

33.

or
or
or

27 m =_f___

f-u

28. m

JlW c,,

-12
._n ;l;
- - = - - - = - -4 -12-u
12ro u
1

12 + u = -48 or u =

or

=_f___

or

- 20
-2=---20-u

_!_=_1__ __!__ = 10 + 1 = 18 = 2_
v 0.8 -1 8
8 4
4
v=-m
9

34. Length of image =

(-f-)b
f-u

1 1 1
35'. -+-=u v f
_du_dv=O
u2 v2

_:66\:m.
or

f-u

f=~m=0.8m,u=-1m

-0.2
-0.2-u

0.2
2 = - - or 0.4+2u=0.2
0.2+u
2u = 0.2-0.4 = -0.2
u = -O.lm

f-u

- -f26. m
2=

921

or

dv

du

-7=~

dv
v 2 du
10x10
_1
_
-=---=----x9rns
_ =-lms 1
dt
u 2 dt
30x30

922

Chapter 23 Ray Optics


42. Clearly, i + r = i + i = 140
or
i = 70

36. Total deviation

(180-2a) + (1'80-2~)

Clearly, plane mirror makes an angle of 20 with vertical


and 70 with horizontal.

= 360-2(a+~)
90-a+90-~+8

But

= 180

43.

!_ = l_
d

8=a+~

or

:. Total deviation = 360-28


d

I-X

37.

x-1

d
f or I = 8f.
u

or

I =-

44.

1--x-v-t-x-v--.j
As is clear from figure, the new distance is 2x - 2v. The
distance of image from object is reduced by an amount 2v

in orie second.
38. Th~ first images is due to r,eflection from the front surface

ie unpolished surface of the mirror. So, only a small


fraction is the incident light energy is reflected. The
second image is due to reflection from polished surface.
So, a major portion of light is reflected. Thus, the second
image is the brightest.
10 em

1---1
0
0'

39.

~I

I'

Observer

1--- x+y-10 ---1


As is clear from figure the distance of image with reference

to observer reduces by 10 em in one second.


40. Clearly, the distance of image from observer is 40 em.
0 ._1_0_e_m-+!i--em-ef

Observer

41.

30 em --~JM

m=_j_

f-u

. If m = + 3, then
-24
3=--24-u

10 em

~~X---!Iiiii--Y-

45.

or

-24- u = -8 or u + 24 = 8 .

or
u = 8 -24 em= -16 em
If m = -3 , then
-3

=--=3.._

-24-U
u+24 = -8
or
u = -32cm.
46. Only the light-gathering power is reduced.
47. Think in terms of rectangular hyperbola.
48. P=(!l-1)(2__2_)
Rl Rz
J..l decreases, p QJ.e.Jreases andf incre~:~ses.
49. Power of the system decreases due to separatidn between
the lenses. So, the focal length increases.
50. Note that two refractive indices are involved. Therefore,
two images will be formed.
51. Diminished erect image is produced only by a concave lens.
52. When convex lens is surrounded by denser medium, it
behaves like a diverging lens.

Chapter 23 Ray Optics

53. P=_!_+_!_-~=0
A !2 fd2

I
I

56.

61. m=--=-2=-3f+u
1
- +U
3

or

or

or

u =~- f

or

= ~ n )'

=-( n~1 )!.tul= n~1 f

-20
.
- = - - - o r -20+u=-40
2 -20+u
u=-40+20 or u=-20cm

I
-25
15 ~10
I= 15 x 2.5 em= 3'7!5 em

-=-

..!.. = (~t-1)(_!_- _!_)


rl

r2

'

r""'

_!_) ~ >Oc,

For lens to be concave, (_!_ _


rl r2 .
. : .
1 1
:T<\:~ . "
or
->- orr1 <r2 .rh ,.
rl r2
~'n
J.)

1 1 1
60. y=~--;]
' 2..

-4

-10

= _R_
2(1!. -1)

or

- - - > 1 or 2(!1-1) < 1


2(!1-1)

or

~t-1 < % or 11 < ( 1 +%)

or
11 < 1.5.
64. Clearly, 2f = 20 em orf = 10 em
Now, u = -15 em, v = ?
F = 10 em
1
1
1
v -15 10
[ 1
1
1 1
1
or
-+-=-or-=--v 15 10
v 10 15
1 3-2 1
or
- = - - = - orv = 30 em
v
30
30
The change in image distance is (30- 20) em ie, 10 em.
65. The power of the given system is a combination of the
positive power of the convex lens, negative power of the
plano-concave lens of water and zero power of the plane
mirror. Clearly, the power of the system decreases.
66. 5 = (1.5

,-=---

2(~t-1)

f f
-f

R
--->R

f+u

59.

Now,f> R

57. m=-'-

v
v=

63. ..!_ = C~t-1)(3_) or

f+u=L
n

1
u = --m = -0.5m
2
1

v
-f
Again, m=-=--=2
u -f/ 2
Clearly, the image is virtual and double the size.

n=-'f+u

1 2

-2u = - +- = 1
3 3

-=---=--

20
!=-em
0.65
= 30.77 em "' 31 em

---2u=3
3

1
1
62. - - - - = v -f/2 f
1 1 2

1 0.65
-=-! 20

58.

or

40

or

55. ..!.. = (1.65 -1)(.3._)

or

The negative sign indicates that the lens is concave.

54. As seen from a rarer medium (L 2 or L3 ) the interface L 1 L 2


is concave and L 2 L3 is convex. The divergence produced
by concave surface is much smaller than the convergence
due to the convex surface. Hence, the arrangement
corresponds to concave-convex lens.

or

or

56-20
d
20x56 -20x56
d = -36 em

1 1
10 4
1 2-5
3
-=--=-! 20
20
20
f = - - em= --6.67 em
-=---

or

1
1
d
20-56 = 20 (-56)

or

1 1
d
.. fl+f2=fd2

923

-1)(*)

..

(i)

... (ii)

924

Chapter 23 Ray Optics

Dividing Eq. (i) by Eq. (ii), we get


_ = 0.5n
5
1.5 -n
or -7.5+5n=0.5n or -7.5=-4.5n
75 5
or
n=-=45 3
67. _

__l.=
(l.5-1)(~-~J
40
R R
1

1
R1

1
R2

1
20

No~' _!_=(~-1](-~J
f
2
20
or

J=- 0;5 (- 210 J

1
1
- = - or f = 80 em
f 80
\
It behave like a convex lens of focal length 80 em.
1 1 1
or

68. - - - = -

v u

1
v

1
-15
1

or

1
10
1

-=--+v
15 10
1 -2+3
-=-v
30
v = 30 em

or
or

76.

;1

69. m=-ff+u
+2=_1__f-10
2f- 20 = f or f = 20 em
20
-2=--- or-40-2u = 20
20+u
-2u = 20 + 40 or u = - 30 em

Now,
or
Again,

or
70. The condition for achromatism is
ffit P1 + WzPz = 0
ffit P1 = -wzPz
W1 = _ Pz
=>
Wz
P1
Now, P1 + P2 = 2D
or

5 + P2 = 2 or P2 = -3D

ffit

-3
5

3
5

-=--=(1)2

_ !11 llz _ !13 _


_ 1
71. zll1 X3 llz X4 113 - - x - - - -4 !11 - llz 113 !14
1!14
72. The central ray goes undeviated. So, J..lz = J..t 1.
Also, J..t3 < J..tz
c
VA
A
73. n1 = - = - = -

v1

vA1

A1

c
VA
A
nz "' - = - = v2

vA 2

A2

]-=(i-1)(~1- ~J
=

(~. - 1)( ~1- ~J

/
(n -1)n'
. 'd'
DlVl mg, - 1 = _:____________:___

fl

or

n-n'

fn'~n -1)
n -n

77. llg sinec = 111 sin 90


or
llg sin ec = 1
When water is poured,
llw sin r = lls sin ec or llw sin r = 1
Again, lla sine = llw sin r
or
lla sine= 1
or
sine = 1 or e = 90
sinr
1
78. -.-:- = tan30= r;::;
smz
v3
sini
r;::;
r;::;
or
- .- =v3 or !l=v3
smr
So, speed of light in Y is .J3 times less.
79.

!1=1=3(1-~)
or

1 1
1
1 2
2
1-- = -,--- or - = 1-- =- or 11 = 11 3 ' 11
3 3
3

Now,--=sinic 2
or
or

. .

sm zc =

. _1(2J
~

or zc = sm

ic = sin- 1(0.67).

[
Chapter 23 Ray Optics
80. Wavelength in vacuum,

3 108
A = X 14
Sx10
= 60ooA
'A

Now: - e x : - 2
' 100 / f J

10 A= 0.6 X 10 A

l2

Again, t ex:

1.5

. 81. M =fo

9 = fo or f. = 9f

'Je

Also, L = fo + fe or 20 = fo + fe
or 20 = 9 fe + f. or 20 = 10 fe
or
fe = 2 em
:. f 0 =9x2cm=18cm
.

82. FortheobJectlve, vo- _

87. Shift

=t(1-~)

1=3(1-.! ) or .!=1-.!_
Jl
3
11
1
1 2
3
or - = 1-- =- or 11 =- = 1.5
Jl
3 3
2
1 1 1

_ =
11 3 8 1 1 4

_!_+ 3.8 = 4 or_!_= 0.2 = .!_


5

or

88.

or

!=--;;--;;_

5
Now:' M 0 =--=
-19
1

3.8
Again, M = M0 X Me

or

= ; + - feD
89. Separat10n
F-10
Je

+D

= 80 + Sx25 = 80 + 125
5+25
30
= 84.16 em= 84.2 em.
90. (1) M =-J0 (1 + f.
fe
D

M=- 200( 1 +~)


5
25

192
u0 =--cm=-1.75cm
110

M = -40 (

84. 'A 2 = 4800 = 0 _8


'A 1 6000
New resolution limit = 0.8 x 0.1 mm = 0.08 mm
3

a= 3.5x10 rad = 3.5 x 180


85 .
3.8x10s
3.8x100
7t
3.5x180x7
38x100x22
400
Also M = fo =
= 40
'
f.
10

1+~ J = -40 x % = -48

(2) M = fo = - 200 = -40


fe
5
91. For reading purposes
u =- 25 em, v =-50 cm,J = ?
1 1 1
1
1
1
-=---= - + - = .. f v u
50 25 so
p = 100 = +2D

f
For distinct vision,
f' = distance of far point = -3m

~= 40x35x180x7 = 2 l.lo"' 21 o.
3Sx100x22
'

P = _!_ = _ _!D =- 0.33 D.

f'

!2

1 1
92. f=-=-m=20cm
p 5

86. tcx:-

d2

j_

1
1
1
100 25
2.5=---- o r - = - - - -0.75 u
u
75 10
1
4 5
1 -8-15
23
-=-- - - o r - = - - = - u
3 2
u
6
6
6
u = --m = -0.26m
23

or

95
-95 =-19 xMeor Me=-=5
19
83. L = v0 + fe -==- v0 = L - f. '
or
V0 =19.2cm
1
1
1
19.2 uo 1.6
1 10 10
--::::::--or
uo 16 192
1 120-10 100
or
192
192
uo
or

fJ

or
t ex: 64
Dividing Eq. (ii) by Eq. (i),
64
16
100t=-=16 or t=-s
4
100

fe
fe

... (i)

!2

'A' = ~ = 6000 A=4oooA


11

1
cx:4
100

or

Now, 11=~
or

925

!2

10

L.

means that the diameter of aperture is


d
'
2

Now,

1
20

---=-

... (ii)

926

Chapter 23 Ray Optics


7t-2A =om

1
1
1
1 1
1
or - - - = - o r - = - - v -25 20
v 20 25
1 5-4
1
1
or
~ = 100 or ~ = 100

2
..

d = lOOcm =1m
or
93. For the eye-piece
ve = -25 cm,f, = 5 em
1
1 1
---=-25 ue 5
1
1 1
1
-1-5
or
-=----or-=--ue
25 5
ue
25
25
or
u =-e
6
25
Now, vo = L-lu e I= 20-

98.

~=sin(~)
.A

sm 2

sin

J2 =--'1..---...=-----.J.
1 . (60+om)
J2
=sm
2
. 45o =sm
. (60o+om)
sm
2

or

120-25
95
- - - e m = - em

=180- 2A

om

om= 30

1
1 1
1
6
Now, - - - - = - or - = - - 1
95 / 6 uo 1
uo 95

. A+om

or

l=--

= 60 + 30 = 90 = 450

or

94. When final image is formed at


M-

1
99. - = ~
2

~:(~)- ~(~)

Now,

x~=-337.5

=+

1 1 1
Now, - = - + F !1 fz
1
1
1
-=-+--

100

so fl -2!1

F= _!_=_!_m=10cm
p

10

or

For image at infinity


M = !?_ =

25

F 10

= 2.5

96. M = fo ,10 = fo ,f0 = 200cm.


f.
20
97.

~=

=-COt = _.!
~
2

! 2 =-2!1

or

13 5

-0.4 2.5
.
95. Power of combination P = P 1 +P2
= + 20- 10

!1
fz

Now, v;; = 16- fe = 16-2.S= 13.5 em


M=

COt

oo ,

or

50f= -2+1 =-1-2!1


2f1
2j1 = SO or f 1 = 25 em

Again,

f 2 = -2 x 25cm = -SOcm

100.

Tij
-2
. A
sm 2

sm
. (A+om)
--

A
2
cot- = ---'-------'. A
sm 2

cos
or

or
or
or

A . (A+om
1
- 2- J

2 = sm

. A
sm2
. (
sm

' . A
sm 2

2- 2
1t

A ) =sm
. (A
- +-om )
2

A A om
---=-+2 2 2
2
Jt

~~A= om
, 2
2

following arguments lead us easily to the right choice.


(i) Angle between any two lines is the same as the angle
between their perpendiculars.
..
(ii)

i=3Q0

1
1.5

sin30
sinr

.
l.S 0 .75
or smr=-=
2
-'
or
r =48.6
(iii) 9 = r- i = 18,6
Required angle = 2 x 18.6 = 37.2

Chapter 23 Ray Optics


101. A = r 1

+ r2

or

or

30 = r 1 + 0

Again,

r 1 = 30

Now: sin i =
' sin30

J2

or

or

. .
{;:;2 1
sml = v.t.X-

or

. .
1
Slnl=

or

or

J2

or

or

ll2 = 1.232
1
sini = --=0.81
c
1.232
ic = sin- 1 0.81

= 54.26
Now, sin 8 = 0 .8 or 8 = 53 .13
This angle is clearly greater than critical angle
corresponding to wavelength 400 run. So, light of 400 nm
wavelength undergoes total internal reflection.
104. For passing the ray from prism,

45

102.

From ,:\,.abc, A+ 90 + (90- i) = 180


or
i =A
Now, complementary angle at point d, 8 = 2 i
e = 2A
Only option (b) satisfies this.

103.

927

..

)l

< cosec

)l

90)
<cosec ( Z

ll <

J2

llmax =

J2

105. When an object is placed between 2f andf (focal length)


of the diverging lens, the image is virtual, erect and
diminished as shown in the graph. To calculate the
distance of the image from the lens, we apply

-1 20+ 0.8 x 1o-14


(400 x 10-9 ) 2

ll1 -

-1 20+
0.8x1o-14
Ill-
400x400x10 18

or

0.8
Ill= 1.20+16
Ill = 1.20 + 0.05

or
or
or

' .'

or

1
1
1 1
u
-20 v 30
(20) (30)
V=
20+30
= - 12 em (to the left of the diverging lens.)

Ill = 1.25
1
. . =--=
0 .8
Slnl
c
1.25

2.

1
v

-=---=>--=---

121.

Chapter 23 Ray Optics

Exercise II
1. As there is no deflection between medium 1 and 2.
Therefore, J.1.1 = J.1.2
2. Imaae can be formed on the screen if it is real. Real image
of reduced size can be formed by a concave mirror or a
convex lens.
Let u = 2/ + x, then
1 1 1
-+-=u

=>

f 2f +x
j(2f + x)
v=
.
f+x

=>

or

-=- x ~

2
orf= 1.20m
0.3

111
x
11x
7. - = - + - - - - =---+----z

!1 !2 fd2

!1 !1 !1

!12

-=-

f+x
j(2f +x)

1-] < l.Om


2f +x+ j(Zf +x) = (2f +x)[1+f+x
,f+x
(2j +X)2 1 0
< . m
f+x
or
(2f +x) 2 < (f +x)
This will be true only whenf < 0.25 m.
3. Given, A = 60

=> f > 0 for every x.

__..
For
X=O,J=oo
Hence, for x = 0, system will behave like a glass plate.
8. We know that

or

11= s~ni and i+r=90


smr
r = 90- i
sini
.
11 =
= tant
sin(90-i)

or

i = tan- 1 (~-t) = tan- 1 (1.62).

It is aiven that u + v = 1.0 m

or

9. Used= f; where,-= C~-t-1)- = (1.5-1)x- = f


R
20 20
=> f= 20 em

i = i'=I' A = 45
4

10. Using equation, the total apparent shift is

i+i'=A+o

or

6. y=~y 1 xh =~16x9 =4x3=12 em.

1 1
1
-=----=

=>

7=(~-1)(0~3)

1
1 1
--+-=2/+x

or

90= 60 +8
() = 30
A

N()te that i = i' is the condition for minimum deviation.


Hence,
8 = 30 = <>min .

4. Image will be real.


We know that
1 1 1
-=--! v u
2'_=1-2'_

2'_=1+m

f.

. s.

v= j(m+1)

!~(.&.-1x]__]_J
1!2
R1 R2

( :. u is negative)

1
1
s=4(1-- )+6(1-- )
4/3
3/2
= 3.0 em
h = h1 + h2 - s = 4 + 6- 3
Thus,
= 7.0 em.
11. We know that
velocity of light in vacuum
11 = velocity of light in water
or

3x1010
4
3 velocity of light in water
=>velocity of light in water = 2.25 x 10 10 cms-1
500 100
= 2.22 X 1o-6 s
Time taken =
X
2.25x1010

Equivalent optical path = Jl x distance travelled in water


= ix500 = 666.64m
3

Chapter 23 Ray Optics


12. Only one converging point is found by this lens. Therefore,
only one image is formed .
13. According to Cartesian sign convention
u = - 40 em, R = - 20 em
l-11 = 1, 112 = 1.33
Applying equation for refraction through spherical
surface, we get
f..lz _ f..l1 = f..lz - f..l1
v
u
R

I
i

929

16. As shown in Fig. (a)

T3: 2 TO
~

(a)

(b)

In this case refraction of the rays starting from t0 takes


d
place from a plane surface. So, we can use dapp = actual
f..l
3=
f..l
4

1.33
1
1.33
v
-40 -20
After solving, v = - 32 em.

or

The magnification is m = hz = f..lJ v


h1 f..lzU

As shown in Fig. (b). In this case refraction takes place

-----= --

h2 = _ __ 1(32)
1.33(-40)
1
h2 = 0.6cm
or
The positive sign shows that the image is erect.

14. This is a modified displacement method problem.


a+d 2
Here, a = 1.8m and - - =a -d 1
Solving we get d = 0.6 m
a2

J.1=:3

or

from a spherical surface. Hence, applying


f..l2 _ f..l1 = f..lz - f..l1
v
u
R
we have,
1
4/3 1-4 / 3
--=--(-25 / 8) -4
-R
1 8
1
1
3R 3 25 75
..
R = 25 em.
Now, to find the focal length we will use the lens maker
formula
-=---=-

or

-dz

!=~
=0.4-m.
15. (i) When angle of prism is 'Small and angle of incidence is
also small, the deviation is given by 8 = (f.1-1)A. Net
deviation by the two prisms is zero. So,
Flint

Crown

81 +82 =0
(~L 1

or

-1)A1 + (f.1 2 -1)A2 = 0.

Here, 111 and 112 are the refractive indices for crown
and flint glasses respectively.
51
4
Hence, f..l1 = 1. +1. g 1.5 and
2
- 1. 77 + 1. 73 -1 75
f..l22
- .
This givesA2

=-

Hence, angle of t1int glass prism is 4. Negative sign


shows that flint glass prism is inverted with respect to
the crown glass prism.
(ii) Net dispersion due to the two prisms is
= (f..l~ - f..l r1 )Al + (f..lbz - f..lr2)A2
= (1.51-1.4g) (6) + (1. 77 -1.73) ( -4)
= -0.04.
:. Net dispersion is- 0.04
t

17. Here, x = u

+v

f
f -v
f+u
f
and image is real, magnification is negative.
f
-(m+1)f
-m=-- u=
f+u'
m
m=--=--

From

f-v

-m=--~v=(m+1)f

Put in Eq.(i)
1
x= - (m + ) f + ( m+ 1)f

Solving, we get,

mx

(m+1) 2
1
1
sini
18. sin C = - = - - - f..l sinr' I sini sinr'
As is clear as shown in figure

LCBD=goo
goo -r +goo -r' = goo

Den~er

r
B

or
sini
:. sin C = _ ___:__
_ _ sini
sin(g0-r) cosr
sini
. ( . )
= - -. =tant : t=r
COSt

C = sin-1 (tani) = sin-\tanr).

Rarer

930

Chapter 23 Ray Optics

19. Let r be the radius of circle through which other objects


become visible. The rays of light must be incident at
critical angle C.

'

. C

24. The combination of two lenses is


1
1
1
As
- =-+F f1
fz

=~= ~rz+hz

sm

_!_ = (J..l1 -1)(2_ + _!_J+ (J..l 2 -1)(J:__- _!_J


F
oo
R
-R oo

J..lzrz = rz + hz

- Jll -1 J..lz -1
--+-R
R

(J..lz -1)rz = hz

_!_ = Jl1- J..lz

h
r=---

~J..lz -1

2r=

~J..lz-1

20. Here, i1 = 60 ,A= 30,8 = 30


As
i1 + i2 -= A+ 8,

i2 = 0
Hence, angle between the ray and the face from which it
emerges = 90-0 = 90.
21. As a convex lens alone can form a real images as well as a
virtual image, therefore, the lens in the present question
is a convex lens. Let f be the focal length of the lens and
m be the magnification produced.
In the first case, when image is real,
u = -16cm, v = (m x 16)cm
1

1
1
1
1 16
- - + - = - o r 1+-=16m 16 f
m
f

.. (i)

"

.. (ii)

f
2
22. When a slab of thickness tis introduced between P and
the mirror, the apparent position of P shifts towards the
Hence, the mirror must be moved in

the same direction through the same distance.


23. In the situation given, the image will be formed at infinity,
if the object is at focus of the lens ie, at 20 em from the
lens. Hence, shift in position of object
= 25- 20 = ( 1 -

5 = (1 - J:__) t
1.5

t = 15 em.

fa

vo

Length of the tube =

22
22
2=- or f=-=11 em.

~}

Uo

1
9 1
-+-=-;v0 =10cm.
vo

Add Eq. (i) and Eq. (ii) we have

uo fe

For objective lens, image is real.

vo

1
1 1
1 6
--+-=-or 1--=-6m 6 f
m f

M=-~_

1
1
1
From-- - = -

From---= -

mirror by ( t-

26. For the relaxed eye, magnifying power is

Given, f a = 1 em.

u = -6cm, v = ( -6m)cm

J..l= sini1 = sin60 = .J3;2 =.J3.


sinr1 sin30
1/2

Vo=+vo,Uo= - ; .

In the second case, when image is virtual,


1

Jl1- J..lz
25. As is clear from figure , A = 30, i1 = 60
As the ray retraces its path on reflection at the silvered face , therefore,
i 2 = 0, r 2 = 0
As
r 1 +r2 =A
r1 + 0 = 30
or
r1 = 30

-45 =-VoX 25' Vo = 9.


uo 5 uo

---=-

As

F=-R-.

or

2h

Diameter

~Jt

V0

+ f, = 10 + 5 = 15 em.

2 7. For total internal reflection at AC face


sini:::: J.lw
J.lg

4
sine::::-3xl.5

. e::::-s
sm
9
28. Note that image formation by a mirror does not depend
on the medium. As P is at a height h above the mirror,
image of P will be at a depth h below the mirror.
If d is depth of liquid in the tank, apparent depth of P,
d-h
Xl=-J..l
:_ Apparent distance between P and its image
d+h d-h 2h
=Xz-Xl = - - - - - = J..l
J..l
J..l

Chapter 23 Ray Optics

931

36. In air

x
lOx
29. c=-v=-tl '
tz

p =

sin C' = _.!_ = ~ =


x x L
11 c
t2
x

. ( 1
f1 = (1.5 -1)
Rl -

1 )
Rz = 2

10

_!_J=
_3_ =4
R
0.5

1J
. _1 (10t
-e ,=Sill

.. . (i)

tz

P'

30. As _.!_ = (!l- 1) (__.!__- __.!__)


f
R1 Rz

Therefore refrective index for A.


then 1.5.

-1

- = - R=-10cm
20 2R'
Refraction from rarer to denser medium

38. Use _.!_ =

_&+llz =llz-111, where u==,v=f


u
v
R
5
0+~= 1. -l =__.!__ f=30cm.
f
10
20'
31. Final image is formed at infinity if the combined focal
length of the two lenses (in contact) becomes 30 em
1
1 1
or
-=-+30 20 f
ie, when another concave lens of focal length 60 em is
kept in contact with the first lens. Similarly, let J..l be the
refractive index of a liquid in which focal length of the
given lens becomes 30 em. Then
210 =(%-1

)(.~1 - ~J

310 = ( 3 2 - 1 )(

~1 - ~J

..
..

(i)

(ii)

From Eqs. (i) and (ii), we get


9
11-- s
32. For convex mirror (positive focal length) image is always
smaller in size. For concave mirror (negative focal length)
image is smaller when object lies beyond 2f.
33. Total internal reflection takes place when ray of light
travels from denser to rarer medium.
8 = -llz and Sill
. 8 = -113 .
Furth er, Sill
13
12
Ill
Ill
113
113
Since,
>
Ill Ill
812 > 813
Smaller the value of critical angle more the chance of
total internal reflection.

34. Using, llz - 111 = llz -Ill , we get


v
u
R
1.5 1.0 1.5 -1.0
20
v
v=60cm
or
'
1
1.
1
35. As !l2':-->-->~2':J2=1.414.
sine sin45 11 -v2
:. Possible value of J..l are 1.5 and 1.6.

= ( 1.5 - 1) X 4 =4.5-4
--x4=0.5D.
4
413

A+B
1
37. !l=~ => ll""~

:. __.!__=(1.5-l)(I__I_)
20
= R
1

In water

4000A will greater

(~
- 1 )[__.!__ - __.!__]
!lm
Rl Rz

Since (IlL< !lm) because lens is of water and IlL= llw


39. !l=-1-=
1
_1__ _
sine sin48 .6 0.75 3
40. Light cannot undergoes total internal reflection when it
is travelling from air to water, ie, from rarer to denser
medium.

1
41. sine=_.!_= - - = = 0.6667
ll 3/2 3
e = sin- 1(0.6667) = 41.8
1 - ,s1n
e=l
42 . From ll = - .
Sill e
!l
As
llv > !lr
ev< e,.
43. As shown in figure
OS= h
When angle of incidence
is slightly greater than
--------e, light undergoes total
------------------
internal reflection.
~-=-=~-=-=-h
c --------~~-=~-=-=
-------:. Diameter of circle of
-----------------------------
light coming from water
----------------surfaee = 2 r = 2 (OB)
------- ~ --s----------
= 2 OS tan e = 2h tan C.
44. In air or water, a convex lens made of glass behaves
as a convergent lens but when it is placed in carbon
disulfide, it behaves as a divergent lens. Therefore, when
a convergent lens is placed inside -a transparent medium
of refractive index greater than that of material of lens, it
behaves as a divergent lens.
It simply concludes that property of a lens whether
the ray is diverging or converging depends on the
surrounding medium.
45. If a mirror is placed in a medium other than air its focal
length does not change asf = R/2, but for 1,the lens

3.

,------~---_-,

_.!._ = Cang -1)(__.!__ _ __.!__)


fa
Rl Rz
__.!__ = C.vng -1)(__.!__ _ __.!__)
fw
Rl Rz
As w ng <0 ng, hence focal length oflens in water increase.
The refractive index of water is 4/3 and that of air is 1.
Hence, llw > lla

and

932

Chapter 23 Ray Optics

46. The magnifying power of telescope in relaxed state is


. ,
m = fo
'
h
.'.s
So, for high magnification, the focal length of objective
length should be larger than of eye-piece.

53. When lens is in air, then


..!_=(aJ.l. -1)(_!_ _J_J.
f
g
' R
R2
'
l

Resolving power of a telescope =

or

_d_:..

1.22A
For high resolving power, diameter (d) of objective should
be higher.
47. We know that power of lens is a reciprocal of its focal
"
1
1
.
= 2D
length, hence P = - =
50
f-

1
' ) (1
1J
~=(1.5-1
--10
R1 R2 .

(;1- ;J=~

or

When lens is in water, then

~f = r:Jlgllw -1J(_!_
_ _!_J
Rl Rz

100

Since, lens is concave hence, its power will be 2D.


If the objective is placed at infinity then
u==,v=?,f=50cm
1 1 1
From the formula, - - - = v
1

I= fo
a

-50
v=-SOcm
Thus, concave lens will form an image of the object at
infinity at a distance of 50 em.
48. When glass surface is made rough, then light incident
on it is scattered in different directions. Due to which its
transparency decreases.
There is no effect of roughness on absorption of light.

.J6 1
.J3 =sine
"
1
" so
smC=-=sm4

fe

-~- = _Q2_ => ~ = so

00

149. Refraction index of diamond w.r.t. liquid zlld = -_,


smC

or

f'=40cm
54. For a telescope

u
1

=(~-1
)x!=_..!._
1.33
5 40

0.5

0.03

55. !tis observed if Li=Le=~LA


4
.
_A+om
and
[=--- .
2
Here,
LA= 60
3
.
and
L t=Le=-LA
4

om = 2x~x60-60
= 30
4
56. Incident ray and finally reflected ray are parallel to each
other means, o= 180

F2

c = 45
50 Red glass transmits only red light and absorbs all the colours
of white light. Thus, when green flower is seen through red
glass it absorbs the green colour, so it appears to be dark.
51. Here, u = -mf, u = v,f = -f
Using lens formula !_..!. = ..!., we have
v u f
or

1
1 ( 1+1) =-(m+1)=-(m+1)
-1
1
-=-v
f
m
mf
u

or

-=--

v
u

From
=>
=>
57.

Jl

-X

:. Coordinate of point Pare (2f, 2f) .


f

oc-

m+1

.
. =v = -1Lmear
magm'fi catiOn
u (m+ 1)
52. The lens formula is
1 1 1
-=--... (i)
! v u
The graph between u and vwill be curve as shown in figure .
Let the coordinates of P be (x, x), then u = -x and v = x
1 1 1
2
- = - - - = - orx = 2f
FromEq.(i),

0=360-20
180 = 360 -20
e = 90

A.

1-lrarer

<

1-ldenser

< Ararer
ie, wavelength decreases.
Actenser

58.

allg

1
1
=-.- -=>sinC=--.
smC
allg

As J..l for violet colour is maximum, so sin C is minimum


and hence critical angle C is minimum for violet colour.
59. As A.r > lvv :. ov > or

The statement given is true neither, in prism spectrum nor


in grating spectrum.

~ I

II

Chapter 23 Ray Optics

. .
60. _!_ =
F

-j

'

C~-t-1) (__!_
- __!_ J
R
R

."',.

0.06

R1

R2

100
3

R
1
Now, -1= - or R 2 = 2R 1
R2 2

100

l
.
71. 8=-,8=1 mm.

R1 =
= 0.045m
200
R 2 = 2R 1 = 2 x 0.045 m = 0.09 m.

So,

and

1
- = ~ = 0.75
4/ 3 4

61. sinC = _!_ = -

70. The angle subtended at the eye becomes 10 times larger.


This happens only when the tree appears 10 times nearer.

11

20
-20
1
1
1
=-+-=40 120 30
4
= -x30 = 40cm

(Taking R1 positive and R2 negative)


1
1
1
-+-=---R1 R2 0.06 X 0.5

. Rz
I

=(~-1)_(-~-~)

1
- - = (1.5 -l)(_!_+_!_J

3
2R1

'

R1

933

1
1t
1
8 = - = - x - rad
60 180 60
1=3m
l
x=r=8
3

= 48

1 = 10 krn.

1t

:. Angle with horizontal = 90- C


= 90 - 48 = 42
62. AI> it is clear from figure distance travelled in. glass place

180 60
72. For dispersion without deviation

81 - 82 = 0, ie , 81 = o2

=DB= OC

cosr

x-

or

Cl-11 -l)Al = C~-tz -l)A2


(1.54 -1)4 = (1.72-1)A2

OB=-t_ .

cosr
90
2

63. AI> the ray turns through 90, therefore, C = - = 45

~-t=-1-=_1_=
sin C

sin 45

lin =J2=1.414.
1/ -v 2

64. p = _!_ = (!l-1)(_!_ _ _!_J


F
R1 R2

= (1.4 -1)(_!_ + 100) = 40 = 2.66D


15
15
00

73. The image formed by a concave mirror is always real


when the object is placed between focal point and infinity.
But when the object is placed between focal point and
pole, then the image is always virtual.
But when the object is virtual ie, object is placed far away
from the focal point, then the image is certainly real.
Hence, the required answer is certainly real if the object
is virtual.
74. Given that,
the refractive index of the lens w.r.t. air, aflg = 1.60 and
the refractive index of water w.r.t. air aflg = 1.33
The focal length of the lens in air, f = 20 em.
We know that for a lens

_!_ =

C~-t-1) (__!___!_ J
R1 R2

When the lens is in the air

66.

~-t=

__!_ = (

sin(A+om)/2 sin(60+60)/2
=
sinA/2
sin60 /2

= sin60 = .J3 = 1. 73
sin30 .2xl / 2
6 7. From the lens formula

!.. -!.. = _!_ = constant

v u f
u is always negative, vis positive.
68. The distance in the experiment are measured by a vernier
scare provided on the microscope .

20

~l

-1)(_!_- __!_
R1

R2 )

or

-=(1.60-1)
1
(1 -1 J
20
R1 R2

or

__!_ = 0.6ox(_!_ _ _!_J

20
R1 __ R2
When the lens is in the wate~

__!_=( 11 -l)(_!_ _ _!_J


f' w g
R1 R2

... (i)

934

Chapter 23 Ray Optics

;.-(:~: 1)(~1 ~2 )

or

-\ = ( a !-lg -a 1-lw
f
a!-lw

or

)(J__J_)
R1

81. Bi-convex lens is cut perpendicularly to the principal axis,


it will become a plano-convex lens.
Focal length of bi-convex lens
]_ = (n

Rz

]__ = (1.60 -1.33 )(__!_ ___!_)


f'
1.33
R1 R2
]__- 27 (__!_ ___!_ )
...(ii)
f' 133 R1 R2
On dividing Eq. (i),by Eq. (ii), we get
.[_ 0 .60x133
20
. 27
or
f' = 20x2 .95cm ~ 60cm.
Hence, its focal length is three times longer than in air.
d
25
75. M=1+-=1+-=6

77. om= A,!J- = 1.5,A =?


sin(A + om)12 sin(A + A)l2
1-1 =
sinAI2
=
sinA12
2 sin A I 2 cos A I 2
sinAI2

3
cos-=-

f=-R-.

(%}A= 2cos-

(%)

For plano-convex lens


]__ = (n

f1

=x(1-~J=6(1- 3 ~ 2 )

!1 =

Comparing Eqs. (i) and (ii), we see that focal length


becomes double.
As power of lens P ex:

1
-,--......,-,---:-

focal length

Hence, power will become half.


New power =

.2 = 2D

82. In normal adjustment,


L = fo + fe = 200 + 4 = 204 em.

1-lv > 1-lr

ov >or
or
D 2 > D1 or D1 > D 2
84. As object is at the centre of the sphere, the image must be
.

.
1
1
8 60
85. From smC=-=--=0.75,C=4.
1-1 1.33
d

=6 x -=2 em
3

Magnification m = fo = 4
fe

fo = 4fe
I

5fe-10 em
fe = 2 em
f a= 8 em
fa = 8 em, fe = 2 em
Hence, L4 and L 1 will be used.

... (ii)

(n-1)

:. Distance of virtual image from centre of sphere = 6 em.

80. Length of tube 10 em


f a + fe = 10 em

or
and

.!..)
=

-1)(]_-

~~~~00~

3 2 =9xi 2 cm
J2 = 1

Putting in Eq. (i),

... (i)

2(n -1)

ie

78. In displacement method,


size of object, 0 = ~1 1 12

79. Shift

~ = cos..

Rz

As 1-lv = 1.525 and 1-lr = 1.520

A
2

83. For o = (!J-- 1)A

3
A
-= 2cos -

R1

-=(n-1)-

or

76. The limit of resolution of an optical instrument arises on


, account of diffraction of light.

-1)(2_- __!__)

= 2 x tan C = 2

x 8 tan 48.6

= 16 x 1.13 = 18.08 em

1inm
-4
86. A=--=5x10
mm
2000
= 5 x 10-7 m = 5000 A
5000
A.'= 2:. =
= 40ooA
1-1
1.25
87. When a ray of light moves from one medium to other,
its velocity changes. This change depends on refractive
index of the medium. Light travels from denser to rater
medium, ie, from medium of higher refractive index to
lower refractive index. So, in second (rarer) medium, its
velocity in.creases.
~

You might also like